You are on page 1of 42

A. Searches and Seizures (Sec. 2, Art. III) 1. Scope of Protection Moncado v.

Peoples Court, 80 Phil 1

Stonehill s. Diokno, 20 SCRA 383

Facts: Respondents issued, on different dates, 42 search warrants against petitioners personally, and/or corporations for which they are officers directing peace officers to search the persons of petitioners and premises of their offices, warehouses and/or residences to search for personal properties books of accounts, financial records, vouchers, correspondence, receipts, ledgers, journals, portfolios, credit journals, typewriters, and other documents showing all business transactions including disbursement receipts, balance sheets and profit and loss statements and Bobbins(cigarettes) as the subject of the offense for violations of Central Bank Act, Tariff and Customs Laws, Internal Revenue Code, and Revised Penal Code. Upon effecting the search in the offices of the aforementioned corporations and on the respective residences of the petitioners, there seized documents, papers, money and other records. Petitioners then were subjected to deportation proceedings and were constrained to question the legality of the searches and seizures as well as the admissibility of those seized as evidence against them. On March 20, 1962, the SC issued a writ of preliminary injunction and partially lifted the same on June 29, 1962 with respect to some documents and papers. Held: a. Search warrants issued were violative of the Constitution and the Rules, thus, illegal or being general warrants. There is no probable cause and warrant did not particularly specify the things to be seized. The purpose of the requirement is to avoid placing the sanctity of the domicile and the privacy of communication and correspondence at the mercy of the whims, caprice or passion of peace officers. b. Document seized from an illegal search warrant is not admissible in court as a fruit of a poisonous tee. However, they could not be returned, except if warranted by the circumstances. c. Petitioners were not the proper party to question the validity and return of those taken from the corporations for which they acted as officers as they are treated as personality different from that of the corporation. Facts: Petitioners, who have prior deportation cases pending, and the corporation they form were alleged to committed "violation of Central Bank Laws, Tariff and Customs Laws, Internal Revenue (Code) and the Revised Penal Code, to which they were served 4 search warrants, directing any peace officer to search petitioners persons and/or premises of their offices, warehouses and/or residences for: books of accounts, financial records, vouchers, correspondence, receipts, ledgers, journals, portfolios, credit journals, typewriters, and other documents and/or papers showing all business transactions including disbursements receipts, balance sheets and profit and loss statements and Bobbins (cigarette wrappers). The items allegedly illegally obtained can be classified into two groups: (1) those found and seized in the offices of aforementioned corporations, and (2) those found in petitioners residences. Petitioners aver that the warrant is illegal for, inter alia: (1) they do not describe with particularity the documents, books and things to be seized; (2) cash money, not mentioned in the warrants, were actually

seized; (3) the warrants were issued to fish evidence against the aforementioned petitioners in deportation cases filed against them; (4) the searches and seizures were made in an illegal manner; and (5) the documents, papers and cash money seized were not delivered to the courts that issued the warrants, to be disposed of in accordance with law x x x. Respondent-prosecutors invoke the Moncado vs Peoples Court ruling: even if the searches and seizures under consideration were unconstitutional, the documents, papers and things thus seized are admissible in evidence against petitioners herein. Issue: Validity of the search warrants. Held: The SC ruled in favor of Stonehill et. al., reversing the Moncado doctrine. Though Stonehill et. al. are not the proper parties to assail the validity of the search warrant issued against their corporation and thus they have no cause of action (only the officers or board members of said corporation may assail said warrant, and that corporations have personalities distinct from petitioners personalities), the 3 w arrants issued to search petitioners residences are hereby declared void. Thus, the searches and seizures made therein are made illegal. The constitution protects the peoples right against unreasonable search and seizure. It provides: (1) that no warrant shall issue but upon probable cause, to be determined by the judge in the manner set forth in said provision; and (2) that the warrant shall particularly describe the things to be seized. In the case at bar, none of these are met. The warrant was issued from mere allegation that petitioners committed a violation of Central Bank Laws, Tariff and Customs Laws, Internal Revenue (Code) and Revised Penal Code. As no specific violation has been alleged, it was impossible for the judges who issued said warrants to have found the existence of probable cause, for the same presupposes the introduction of competent proof that the party against whom it is sought has performed or committed violations of the law. In other words, it would be a legal heresy, of the highest order, to convict anybody of a violation of Central Bank Laws, Tariff and Customs Laws, Internal Revenue (Code) and Revised Penal Code, as alleged in the aforementioned applications without reference to any determinate provision of said laws or codes. General warrants are also to be eliminated, as the legality or illegality of petitioners transactions is immaterial to the invalidity of the general warrant that sought these effects to be searched and seized: Books of accounts, financial records, vouchers, journals, correspondence, receipts, ledgers, portfolios, credit journals, typewriters, and other documents and/or papers showing all business transactions including disbursement receipts, balance sheets and related profit and loss state ments. The Court also holds that the only practical means of enforcing the constitutional injunction against unreasonable searches and seizures is, in the language of the Federal Supreme Court: x x x If letters and private documents can thus be seized and held and used in evidence against a citizen accused of an offense, the protection of the 4th Amendment, declaring his rights to be secure against such searches and seizures, is of no value, and, so far as those thus placed are concerned, might as well be stricken from the Constitution. The efforts of the courts and their officials to bring the guilty to punishment, praiseworthy as they are, are not to be aided by the sacrifice of those great principles established by years of endeavor and suffering which have resulted in their embodiment in the fundamental law of the land.
Only party affected may contest legality of seizure effected by search warrants. officers of certain corporations, from which documents, papers and things were seized by means of search warrants, have no cause of action to assail the legality of the seizures because said corporations have personalities distinct and separate from those said officers. The legality of a seizure can be contested only by the party whose right have been impaired thereby. The objection to an unlawful search is purely personal and cannot be availed of by third person. When illegally seized evidence is admissible. officers of certain corporations cannot validly object to the use in evidence against them of the documents, papers and things seized from the offices and premises of the corporations

since the right to object to their admission in evidence belongs exclusively to the corporations, to which the seized effects belong, and may not be invoked by the corporate officers in proceedings against them in their individual capacity.

Alih v. Castro, 151 SCRA 279 Facts: Respondents who were members of the Philippine marine and defense forces raided the compound occupied by petitioner in search of loose firearms, ammunitions and explosives. A shoot-out ensued after petitioners resisted the intrusion by the respondents, killing a number of men. The following morning, the petitioners were arrested and subjected to finger printing, paraffin testing and photographing despite their objection. Several kinds of rifle, grenades and ammunitions were also confiscated. The petitioners filed an injunction suit with a prayer to have the items illegally seized returned to them and invoked the provisions on the Bill of Rights The respondents admitted that the operation was done without a warrant but reasoned that they were acting under superior orders and that operation was necessary because of the aggravation of the peace and order problem due to the assassination of the city mayor. Issue: Whether or not the seizing of the items and the taking of the fingerprints and photographs of the petitioners and subjecting them to paraffin testing are violative of the bill of Rights and are inadmissible as evidence against them. Held: The court held that superior orders nor the suspicion that the respondents had against petitioners did not excuse the former from observing the guaranty provided for by the constitution against unreasonable searches and seizure. The petitioners were entitled to due process and should be protected from the arbitrary actions of those tasked to execute the law. Furthermore, there was no showing that the operation was urgent nor was there any showing of the petitioners as criminals or fugitives of justice to merit approval by virtue of Rule 113, Section 5 of the Rules of Court. The items seized, having been the fruits of the poisonous tree were held inadmissible as evidence in any proceedings against the petitioners. The operation by the respondents was done without a warrant and so the items seized during said operation should not be acknowledged in court as evidence. But said evidence should remain in the custody of the law (custodia egis). However, as to the issue on finger-printing, photographing and paraffin-testing as violative of the provision against self-incrimination, the court held that the prohibition against self-incrimination applies to testimonial compulsion only. As Justice Holmes put it in Holt v. United States, 18 The prohibition of compelling a man in a criminal court to be a witness against himself is a prohibition of the use of physical or moral compulsion to extort communications from him, not an exclusion of his body as evidence when it may be material. Raid without search warrant; Constitutional precept that the civilian authority is at all times supreme over the military, defied in case at bar when the military proceeded to make the raid without a search warrant. In acting as they did, they also defied the precept that the civilian authority is at all times supreme over the military so clearly proclaimed in the 1973 Constitution. In the instant case, the respondents simply by-passed the civil courts, which had the authority to determine whether or not there was probable cause to search the

petitioners premises. Instead, they proceeded to make the raid without a search warrant on their ow n unauthorized determination of the petitioners guilt.
2. Meaning of search Katz v. United States, 389 U.S. 347 (1967)
Charles Katz used a public pay phone booth to transmit illegal gambling wagers from Los Angeles to Miami and Boston. Unknown to Katz, the FBI was recording his conversations via an electronic eavesdropping device attached to the exterior of the phone booth. Katz was convicted based on these recordings. He challenged his conviction, arguing that the recordings were obtained in violation of his Fourth Amendment rights. The Court of Appeals sided with the FBI because there was no physical intrusion into the phone booth itself. The Supreme Court granted certiorari.

Ruling

"The Government's activities in electronically listening to and recording the petitioner's words violated the privacy upon which he justifiably relied while using the telephone booth and thus constituted a 'search and seizure' within the meaning of the Fourth Amendment (is the part of the Bill of Rights which guards against unreasonable searches and seizures, along with requiring any warrant to be judicially sanctioned and supported by probable cause). " Justice Stewart [1] Regardless of the location, a conversation is protected from unreasonable search and seizure under the Fourth Amendment if it is made with a reasonable expectation of privacy. Wiretapping counts as a search (physical intrusion is not necessary).
Search and seizure is a legal procedure used in many civil law and common law legal systems whereby police or other authorities and their agents, who suspect that a crime has been committed, do a search of a person's property and confiscate any relevant evidence to the crime. Search and seizure (including arrest) should be limited in scope according to specific information supplied to the issuing court, usually by a law enforcement officer, who has sworn by it.

3. Requisites of a Valid Search Warrant Existence of Probable Cause Burgos v. Chief of Staff, 133 SCRA 800

Doctrine: A machinery which is movable by nature becomes immobilized when placed by the owner of the tenement, property or plant, but not so when placed by a tenant, usufructuary, or any other person having only temporary right, unless such person acted as the agent of the owner. Facts: Armed with a search warrant issued by the Court of First Instance of Rizal, law enforcement officers searched the offices of the We forum and Metropolitan Mail newspapers. During the course of the search, the law enforcement officers seized office and printing machines, equipment, paraphernalia and several other materials used in the distribution of newspapers. It is contended by the petitioners that the above mentioned documents could not have provided sufficient basis for the finding of a probable cause upon which a warrant may validly issue in accordance with sec. 3, art. 4 of the 1973 Cons. Which provides: xxx and no search warrant or warrant of
arrest shall issue except upon probable cause to be determined by the judge, or such other responsible officer as may be authorized by law, after examination under oath or affirmation of the complainant and the witnesses he may produce, and particularly describing the place to be searched and the person and thing to be seized.

Petitioner avers, among others, that the seizure of the properties mentioned above amounts to seizure of real properties, which cannot be validly conducted under the strength of a search warrant. It must be noted that real properties are not susceptible of confiscation under a search warrant. Hence this appeal which assails the validity of the search and the seizure of the properties of the petitioner. Issue: Whether there is merit in the petitioners assertion that real property were invalidly seized under the disputed warrants. Held: No. The petitioners assertion does not hold water. Under Article 415(5) of the civil code, machinery, receptacles, instruments or implements intended by the owner of the tenement for an industry or works which may be carried on in a building or on a piece of land and which tend directly to meet the needs of the said industry or works are considered immovable property. In another case decided by the Court, in which the abovementioned legal provision was invoked, it was ruled that machinery which is movable by nature becomes immobilized when placed by the owner of the tenement, property or plant, but not so when placed by a tenant, usufructuary, or any other person having only temporary right, unless such person acted as the agent of the owner. In the case at bar, petitioners do not claim to be the owners of the land and/or building on which the machineries were placed. This being the case, the machineries in question, while in fact bolted to the ground remains movable property susceptible to seizure under a search warrant.

Probable cause for a search is defined as such facts and circumstances which would lead a reasonably discreet and prudent man to believe that an offense has been committed and that the objects sought in connection with the offense are in the place sought to be searched.
The issued search warrants by the respondent judge are null and void and are set aside. The prayer for writ of mandatory injunction for the return of the seized articles is hereby granted and all articles seized thereunder are hereby ordered released to the petitioners.
Stonehill v. Diokno, supra

Facts: Petitioners, who have prior deportation cases pending, and the corporation they form were alleged to committed "violation of Central Bank Laws, Tariff and Customs Laws, Internal Revenue (Code) and the Revised Penal Code, to which they were served 4 search warrants, directing any peace officer to search petitioners persons and/or premises of their offices, warehouses and/or residences for: books of accounts, financial records, vouchers, correspondence, receipts, ledgers, journals, portfolios, credit journals, typewriters, and other documents and/or papers showing all business transactions including disbursements receipts, balance sheets and profit and loss statements and Bobbins (cigarette wrappers). The items allegedly illegally obtained can be classified into two groups: (1) those found and seized in the offices of aforementioned corporations, and (2) those found in petitioners residences. Petitioners aver that the warrant is illegal for, inter alia: (1) they do not describe with particularity the documents, books and things to be seized; (2) cash money, not mentioned in the warrants, were actually seized; (3) the warrants were issued to fish evidence against the aforementioned petitioners in deportation cases filed against them; (4) the searches and seizures were made in an illegal manner; and (5) the documents, papers and cash money seized were not delivered to the courts that issued the warrants, to be disposed of in accordance with law x x x. Respondent-prosecutors invoke the Moncado vs Peoples Court ruling: even if the searches and seizures under consideration were unconstitutional, the documents, papers and things thus seized are admissible in evidence against petitioners herein.

Issue: Validity of the search warrants. Held: The SC ruled in favor of Stonehill et. al., reversing the Moncado doctrine. Though Stonehill et. al. are not the proper parties to assail the validity of the search warrant issued against their corporation and thus they have no cause of action (only the officers or board members of said corporation may assail said warrant, and that corporations have personalities distinct from petitioners personalities), the 3 warrants issued to search petitioners residences are hereby declared void. Thus, the searches and seizures made therein are made illegal. The constitution protects the peoples right against unreasonable search and seizure. It provides: (1) that no warrant shall issue but upon probable cause, to be determined by the judge in the manner set forth in said provision; and (2) that the warrant shall particularly describe the things to be seized. In the case at bar, none of these are met. The warrant was issued from mere allegation that petitioners committed a violation of Central Bank Laws, Tariff and Customs Laws, Internal Revenue (Code) and Revised Penal Code. As no specific violation has been alleged, it was impossible for the judges who issued said warrants to have found the existence of probable cause, for the same presupposes the introduction of competent proof that the party against whom it is sought has performed or committed violations of the law. In other words, it would be a legal heresy, of the highest order, to convict anybody of a violation of Central Bank Laws, Tariff and Customs Laws, Internal Revenue (Code) and Revised Penal Code, as alleged in the aforementioned applications without reference to any determinate provision of said laws or codes. General warrants are also to be eliminated, as the legality or illegality of petitioners transactions is immaterial to the invalidity of the general warrant that sought these effects to be searched and seized: Books of accounts, financial records, vouchers, journals, correspondence, receipts, ledgers, portfolios, credit journals, typewriters, and other documents and/or papers showing all business transactions including disbursement receipts, balance sheets and related profit and loss statements. The Court also holds that the only practical means of enforcing the constitutional injunction against unreasonable searches and seizures is, in the language of the Federal Supreme Court: x x x If letters and private documents can thus be seized and held and used in evidence against a citizen accused of an offense, the protection of the 4th Amendment, declaring his rights to be secure against such searches and seizures, is of no value, and, so far as those thus placed are concerned, might as well be stricken from the Constitution . The efforts of the courts and their officials to bring the guilty to punishment, praiseworthy as they are, are not to be aided by the sacrifice of those great principles established by years of endeavor and suffering which have resulted in their embodiment in the fundamental law of the land.
Asian Surety v. Herrera, 54 SCRA 312

I. Facts of the case:19611964 Believed that the four offenses alleged to Asian Surety & Insurance Co., Inc. - estafa, falsification, tax evasion and insurance fraud took place. October 27, 1965 Judge Herrera, upon the sworn application of NBI agent Celso Zoleta, Jr. supported by the deposition of his witness, Manuel Cuaresma, issued a search warrant against the corporation. Armed with the search warrant, Zoleta and other agents entered the premises of the Republic Supermarket Building and served the search warrant in the presence of Mr. William Li Yao, president and chairman of the board of directors of the insurance firm. After the search they seized and carried away two carloads of documents, papers and receipt.

The Asian Surety & Insurance Company Inc. filed a petition assailing the validity of the search warrant, claiming that it was issued in contravention of the explicit provisions of the Constitution and the Rules of Court, particularly Section 3, of Art. IV of the New Constitution, and Sections 3, 5, 8 and 10 of Rule 126 of the Rules of Court. II. Issue of the Case The search warrant reads in part: x x x property should be seized and brought to the undersigned. The judge made no attempt to determine whether the property he authorized to be searched and seized pertains specifically to any one of the three classes of personal property that may be searched and seized under a search warrant. It appears that the respondent judge intended the search to apply to all the three classes of property. The petitioner also manifested that the seizure of TWO carloads of their papers has paralyzed their business to the grave prejudice of not only the company, its workers, agents, employees but also of its numerous insured and beneficiaries of bonds issued by it, including the government itself, and of the general public. Asian Surety & Insurance Co., Inc. contests the validity of the search warrant on the ground that it authorized the search and seizure of personal properties so vaguely described and not particularized. It clearly violates the Constitutional Law: the right to be secure against unreasonable searches and seizures stating that the right of personal security, and that involves the exemption of his private affairs, books and papers from the inspection and scrutiny of others, is an important factor to peace and happiness. It also assails the non-compliance with the Sec 2 of Rule 126 which provides that a search warrant may be issued for the search and seizure of the enumerated personal properties including the property subject of the offense. III. Ratio Decidendi A. Lower Court Judge Herrera approved the respondents application and commanded them to make an immediate search at any time to Asian Surety & Insurance Co., Inc. for there are good & sufficient reasons to believe that Mr. Yao or his employees committed offenses using their properties. B. Supreme Court The petition was granted; the search warrant of October 27,1965, is nullified and set aside, and the respondents were ordered to return immediately all documents, papers and other objects seized or taken thereunder. Without costs.

Castro v. Pabalan, 70 SCRA 477 Facts: Judge Pabalan ordered the issuance of a search warrant despite failure of the application of Lumang or the warrant itself to specify the offense, to examine the applicant as well as his witnesses on the part of the Judge, and to describe with particularity the place to be searched and the things to be seized. Judge never refuted the assertions when required to answer. Application alleged that applicants wee informed and claimed that they verified the report that Maria Castro and Co Ling are in possession of narcotics and other contraband in Barrio Padasil, Bangar, La Union without specifying the particular place in the Barrio. No complete description of the goods and inquiry was brief. Upon actual search, it turned out that it was in Barrio Ma. Cristina and not in Padasil. Issue: Whether or not the search warrant is validly issued. Held: Search warrant issued illegal for violation of the 1935 Constitution and the Rules of Court because the two basic requirements are not complied with: (a) no warrant shall issue but upon probable cause, (b) the warrant shall particularly

describe the things to be seized, thus, a general warrant. However, things seized cannot be returned and shall be destroyed, except the liquors, playing cards, distilled water and five bottles of Streptomycin. Effect of irregularity in the issuance of search warrant the illegality of the search warrant does not call for the return of the thing seized, the possession of which is prohibited by the law. This is the established doctrine in this jurisdiction. Pita v. CA, 178 SCRA 362

Facts: In 1983, elements of the Special Anti-Narcotics Group, and the Manila Police, seized and confiscated from dealers along Manila sidewalks, magazines believed to be obscene. These were later burned. One of the publications was Pinoy Playboy published by Leo Pita. He filed an injunction case against the mayor of manila to enjoin him from confiscating more copies of his magazine and claimed that this was a violation of freedom of speech. The court ordered him to show cause. He then filed an Urgent Motion for issuance of a temporary restraining order against indiscriminate seizure. Defendant Mayor Bagatsing admitted the confiscation and burning of obscence reading materials but admitted that these were surrendered by the stall owners and the establishments were not raided. The other defendant, WPD Superintendent, Narcisco Cabrera, filed no answer. On January 11, 1984, the trial court issued an Order setting the case for hearing on January 16, 1984 "for the parties to adduce evidence on the question of whether the publication 'Pinoy Playboy Magazine alleged (sic) seized, confiscated and/or burned by the defendants, are obscence per se or not". On February 3, 1984, the trial court promulgated the Order appealed from denying the motion for a writ of preliminary injunction, and dismissing the case for lack of merit The CA also dismissed the appeal due to the argument that freedom of the press is not without restraint. In the SC, the petitioner claimed that: 1. The CA erred in holding that the police officers could without any court warrant or order seize and confiscate petitioner's magazines on the basis simply of their determination that they are obscene. 2. The Court of Appeals erred in affirming the decision of the trial court and, in effect, holding that the trial court could dismiss the case on its merits without any hearing thereon when what was submitted to it for resolution was merely the application of petitioner for the writ of preliminary injunction. Issue: Was the seizure constitutional? Held: No. Petition granted Ratio: Test for obscenity: "whether the tendency of the matter charged as obscene, is to deprave or corrupt those whose minds are open to such immoral influences and into whose hands a publication or other article charged as being obscene may fall. Also, "whether a picture is obscene or indecent must depend upon the circumstances of the case, and that ultimately, the question is to be decided by the "judgment of the aggregate sense of the community reached by it." (Kottinger)

When does a publication have a corrupting tendency, or when can it be said to be offensive to human sensibilities? The issue is a complicated one, in which the fine lines have neither been drawn nor divided. Katigbak- "Whether to the average person, applying contemporary standards, the dominant theme of the material taken as a whole appeals to prurient interest." Kalaw-Katigbak represented a marked departure from Kottinger in the sense that it measured obscenity in terms of the "dominant theme" of the work, rather than isolated passages, which were central to Kottinger (although both cases are agreed that "contemporary community standards" are the final arbiters of what is "obscene"). Kalaw-Katigbak undertook moreover to make the determination of obscenity essentially a judicial question and as a consequence, to temper the wide discretion Kottinger had given unto law enforcers. The latest say on American jurisprudence was Miller v. California, which expressly abandoned Massachusettes, and established "basic guidelines," to wit: "(a) whether 'the average person, applying contemporary standards' would find the work, taken as a whole, appeals to the prurient interest . . .; (b) whether the work depicts or describes, in a patently offensive way, sexual conduct specifically defined by the applicable state law; and (c) whether the work, taken as a whole, lacks serious literary, artistic, political, or scientific value. The lack of uniformity in American jurisprudence as to what constitutes "obscenity" has been attributed to the reluctance of the courts to recognize the constitutional dimension of the problem. Apparently, the courts have assumed that "obscenity" is not included in the guaranty of free speech, an assumption that, as we averred, has allowed a climate of opinions among magistrates predicated upon arbitrary, if vague theories of what is acceptable to society. In the case at bar, there is no challenge on the right of the State, in the legitimate exercise of police power, to suppress smut provided it is smut. For obvious reasons, smut is not smut simply because one insists it is smut. So is it equally evident that individual tastes develop, adapt to wide-ranging influences, and keep in step with the rapid advance of civilization. What shocked our forebears, say, five decades ago, is not necessarily repulsive to the present generation. But neither should we say that "obscenity" is a bare (no pun intended) matter of opinion. As we said earlier, it is the divergent perceptions of men and women that have probably compounded the problem rather than resolved it. Undoubtedly, "immoral" lore or literature comes within the ambit of free expression, although not its protection. In free expression cases, this Court has consistently been on the side of the exercise of the right, barring a "clear and present danger" that would warrant State interference and action. But the burden to show this lies with the authorities. "There must be objective and convincing, not subjective or conjectural, proof of the existence of such clear and present danger." As we so strongly stressed in Bagatsing, a case involving the delivery of a political speech, the presumption is that the speech may validly be said. The burden is on the State to demonstrate the existence of a danger, a danger that must not only be: (1) clear but also, (2) present, to justify State action to stop the speech. The Court is not convinced that the private respondents have shown the required proof to justify a ban and to warrant confiscation of the literature for which mandatory injunction had been sought below. First of all, they were not possessed of a lawful court order: (1) finding the said materials to be pornography, and (2) authorizing them to carry out a search and seizure, by way of a search warrant. Has petitioner been found guilty for publishing obscene works under Presidential Decrees Nos. 960 and 969? This not answered, one can conclude that the fact that the former respondent Mayor's act was sanctioned by "police power" is no license to seize property in disregard of due process. The PDs dont giv e the authorities the permission to execute high-handed acts. It is basic that searches and seizures may be done only through a judicial warrant, otherwise, they become unreasonable and subject to challenge.

There is of course provision for warrantless searches under the Rules of Court but as the provision itself suggests, the search must have been an incident to a lawful arrest and it must be on account fo a crime committed. The Court rejected the argument that "[t]here is no constitutional nor legal provision which would free the accused of all criminal responsibility because there had been no warrant, and there is no "accused" here to speak of, who ought to be "punished". Second, to say that the respondent Mayor could have validly ordered the raid (as a result of an anti-smut campaign) without a lawful search warrant because, in his opinion, "violation of penal laws" has been committed, is to make the respondent Mayor judge, jury, and executioner rolled into one. The court is not convinced that the private respondents have shown the required proof to justify a ban and to warrant confiscation of the literature for which mandatory injunction had been sought below. First of all, they were not possessed of a lawful court order: 1) finding the said materials to be pornography, and 2) authorizing them to carry out a search and seizure, by way of a search warrant
Pp v. Salanguit, G.R. No. 133254-55, April 19, 2001

Facts: On 26 December 1995, Sr. Insp. Aguilar applied for a warrant in the Regional Trial Court, Branch 90, Dasmariias, Cavite, to search the residence of Robert Salanguit y Ko on BinhaganSt., Novaliches, Quezon City. He presented as his witness SPO1 Edmund Badua, who testified that as a poseur-buyer, he was able to purchase2.12 grams of shabu from Salanguit. The sale took place in Salunguits room, and Badua saw that the shabu was taken by Salunguit from a cabinet inside his room. The application was granted, and a search warrant was later issued by Presiding Judge Dolores L. Espaol. At about10:30 p.m. of said day, a group of about 10policemen, along with one civilian informer, went to the residence of Salunguit to serve the warrant. The police operatives knocked on Salanguits door, but nobody opened it. They heard people inside the house, apparently panicking. The police operatives then forced the door open and entered the house. After showing the search warrant to the occupants of the house, Lt. Cortes and his group started searching the house. They found 12 small heat-sealed transparent plastic bags containing a white crystalline substance, a paper clip box also containing a white crystalline substance, and two bricks of dried leaves which appeared to be marijuana wrapped in newsprint having a total weight of approximately 1,255 grams. A receipt of the items seized was prepared, but Salanguit refused to sign it. After the search, the police operatives took Salanguit with them to Station 10, EDSA, Kamuning, Quezon City,along with the items they had seized. PO3Duazo requested a laboratory examination of the confiscated evidence. The white crystalline substance with a total weight of 2.77 grams and those contained in a small box with a total weight of 8.37 grams were found to be positive for methamphetamine hydrochloride. On the other hand, the two bricks of dried leaves, one weighing 425 grams and the other 850 grams, were found to be marijuana. Charges against Roberto Salanguit y Ko for violations of Republic Act (RA) 6425, i.e. for possession of shabu and marijuana, (Criminal Cases Q-95-64357 and Q-9564358, respectively) were filed on 28December 1995. After hearing, the trial court rendered its decision, convicting Salanguit in Criminal Cases Q-95-64357 and Q-95-64358 for violation of Section 16 and 8, respectively, RA6425, and sentencing him to suffer an indeterminate sentence with a minimum of 6months of arresto mayor and a maximum of 4years and 2 months of prision correccional, and reclusion perpetua and to pay a fine of P700,000.00, respectively. Salanguit appealed; contesting his conviction on the grounds that(1) the admissibility of the shabu allegedly recovered from his residence as evidence against him on the ground

that the warrant used in obtaining it was invalid; (2) the admissibility in evidence of the marijuana allegedly seized from Salanguit to the plainview doctrine; and (3) the employment of unnecessary force by the police in the execution of the warrant. Issue: Whether the warrant was invalid for failure of providing evidence to support the seizure of drug paraphernalia, and whether the marijuana may be included as evidence in light of the plain view doctrine. Held: The warrant authorized the seizure of undetermined quantity of shabu and drug paraphernalia. Evidence was presented showing probable cause of the existence of methamphetamine hydrochloride or shabu. The fact that there was no probable cause to support the application for the seizure of drug paraphernalia does not warrant the conclusion that the search warrant is void. This fact would be material only if drug paraphernalia was in fact seized by the police. The fact is that none was taken by virtue of the search warrant issued. If at all, therefore, the search warrant is void only insofar as it authorized the seizure of drug paraphernalia, but it is valid as to the seizure of methamphetamine hydrochloride as to which evidence was presented showing probable cause as to its existence. In sum, with respect to the seizure of shabu from Salanguits residence, Search Warrant 160 was properly issued, such warrant being founded on probable cause personally determined by the judge under oath or affirmation of the deposing witness and particularly describing the place to be searched and the things to be seized. With respect to, and in light of the plain view doctrine, the police failed to allege the time when the marijuana was found, i.e., whether prior to, or contemporaneous with, the shabu subject of the warrant, or whether it was recovered on Salanguits person or in an area within his immediate control. Its recovery, therefore, presumably during the search conducted after the shabu had been recovered from the cabinet, as attested to by SPO1 Badua in his deposition, was invalid. Thus, the Court affirmed the decision as to Criminal Case Q-95-64357 only.

Personal Determination of PC by the judge Placer v. Villanueva, 126 SCRA 463

Facts: Petitioners filed information in the city court and they certified that Preliminary Investigation and Examination had been conducted and that prima facie cases have been found. Upon receipt of said information, respondent judge set the hearing of the criminal cases to determine propriety of issuance of warrants of arrest. After the hearing, respondent issued an order requiring petitioners to submit to the court affidavits of prosecution witnesses and other documentary evidence in support of the information to aid him in the exercise of his power of judicial review of the findings of probable cause by petitioners. Petitioners petitioned for certiorari and mandamus to compel respondent to issue warrants of arrest. They contended that the fiscals certification in the information of the existence of probable cause constitutes sufficient justification for the judge to issue warrants of arrest. Issue: Whether or Not respondent city judge may, for the purpose of issuing warrants of arrest, compel the fiscal to submit to the court the supporting affidavits and other documentary evidence presented during the

preliminary investigation.

Held: Judge may rely upon the fiscals certification for the existence of probable cause and on the basis thereof, issue a warrant of arrest. But, such certification does not bind the judge to come out with the warrant. The issuance of a warrant is not a mere ministerial function; it calls for the exercise of judicial discretion on the part of issuing magistrate. Under Section 6 Rule 112 of the Rules of Court, the judge must satisfy himself of the existence of probable cause before issuing a warrant of arrest. If on the face of the information, the judge finds no probable cause, he may disregard the fiscals certification and require submission of the affidavits of witnesses to aid him in arriving at the conclusion as to existence of probable cause. Petition dismissed. We hold that respondent did not abuse his discretion in doing so. From the informations and affidavits presented to him, he found the charges patently without basis or merit. For the respondent to issue the warrants of arrest and try the accused would only expose the latter to unnecessary harassment, anxiety and expense. And as already pointed out, under the Rule on Summary Procedure in Special Cases, the respondent judge has the power to order the outright dismissal of the charge if, from the information and the affidavits attached thereto, he finds the same to be patently without basis or merit.

Soliven v. Makasiar, supra


Ho v. Pp, 280 SCRA 365

The Facts Both petitions have the same factual backdrop. On August 8, 1991, the Anti-Graft League of the Philippines, represented by its chief prosecutor and investigator, Atty. Reynaldo L. Bagatsing, filed with the Office of the Ombudsman a complainti[2] against Doris Teresa Ho, Rolando S. Narciso (petitioners in G.R. Nos. 106632 and 106678, respectively), Anthony Marden, Arsenio Benjamin Santos and Leonardo Odoo. The complaint was for alleged violation of Section 3 (g) of Republic Act 3019ii[3] prohibiting a public officer from entering into any contract or transaction on behalf of the government if it is manifestly and grossly disadvantageous to the latter, whether or not the public officer profited or will profit thereby. After due notice, all respondents therein filed their respective counter-affidavits with supporting documents. On January 8, 1992, Graft Investigation Officer Titus P. Labrador (hereafter, GIO Labrador) submitted his resolutioniii[4] with the following recommendations: WHEREFORE, all premises considered, it is respectfully recommended that an information for violation of Section 3 (g) of R.A. 3019 as amended be filed against respondent Rolando S. Narciso before the Sandiganbayan. It is likewise recommending that the case against the other respondents be DISMISSED for insufficiency of evidence. However, after a review of the above resolution, Special Prosecution Officer Leonardo P. Tamayo (hereafter, SPO Tamayo) recommended that both Rolando Narciso and Doris Teresa Ho be charged wit h violation of

Section 3 (e) of R.A. 3019. The resolution of GIO Labrador, as modified by the memorandumiv[5] of SPO Tamayo, was approved by Ombudsman Conrado M. Vasquez on May 5, 1992. Thus, herein petitioners were charged accordingly before the Sandiganbayan in an informationv[6] filed on May 18, 1992. Attached to the information were the resolution of GIO Labrador and the memorandum of SPO Tamayo. The said information reads: The undersigned Special Prosecution Officer III, Office of the Special Prose cutor, hereby accuses ROLANDO NARCISO and DORIS TERESA HO, President of National Marine Corporation, of violation of Section 3(e) of RA 3019, as amended, committed as follows: That on or about April 4, 1989, and for sometime prior and/or subsequent thereto, in the City of Manila, Philippines and within the jurisdiction of this Honorable Court, the above-named accused ROLANDO NARCISO, a public officer, being then the Vice-President of the National Steel Corporation (NSC), a government-owned or controlled corporation organized and operating under the Philippine laws, and DORIS TERESA HO, a private individual and then the President of National Marine Corporation (NMC), a private corporation organized and operating under our Corporation law, conspiring and confederating with one another, did then and there wilfully, unlawfully and criminally, with evident bad faith and through manifest partiality, cause undue injury to the National Steel Corporation (NSC), by entering without legal justification into a negotiated contract of affreightment disadvantageous to the NSC for the haulage of its products at the rate of P129.50/MT, from Iligan City to Manila, despite their full knowledge that the rate they have agreed upon was much higher than those offered by the Loadstar Shipping Company, Inc. (LSCI) and Premier Shipping Lines, Inc. (PSLI), in the amounts of P109.56 and P123.00 per Metric Ton, respectively, in the public bidding held on June 30, 1988, thereby giving unwarranted benefits to the National Marine Corporation, in the total sum of One Million One Hundred Sixteen Thousand Fifty Two Pesos and Seventy Five Centavos (P1,116,052.75), Philippine Currency, to the pecuniary damage and prejudice of the NSC in the aforestated sum. The said offense was committed by Rolando S. Narciso in the performance of his official functions as Vice-President of the National Steel Corporation. CONTRARY TO LAW. Acting on the foregoing information, the Sandiganbayan issued the now questioned warrant of arrest against Petitioners Ho and Narciso. Petitioner Ho initially questioned the issuance thereof in an Urgent Motion to Recall Warrant of Arrest/Motion for Reconsideration which was adopted by Petitioner Narciso. They alleged that the Sandiganbayan, in determining probable cause for the issuance of the warrant for their arrest, merely relied on the information and the resolution attached thereto, filed by the Ombudsman without other supporting evidence, in violation of the requirements of Section 2, Article III of the Constitution, and settled jurisprudence. Respondent Sandiganbayan denied said motion in the challenged Resolution. It ratiocinated in this wise: Considering, therefore, that this Court did not rely solely on the certification appearing in the information in this case in the determination of whether probable cause exists to justify the issuance of the warrant of arrest but also on the basis predominantly shown by the facts and evidence appearing in the resolution/memorandum of responsible investigators/ prosecutors, then the recall of the warrant of arrest, or the reconsideration sought for, cannot be granted. More so, when the information, as filed, clearly shows that it is sufficient in form and substance based on the facts and evidence adduced by both parties during the preliminary investigation. To require this Court to have the entire record of the preliminary investigation to be produced before it, including the evidence submitted by the complainant and the accused-respondents, would appear to be an exercise in futility.

Thus, these petitions. The Issue Petitioner Ho raises this sole issue: May a judge determine probable cause and issue *a+ warrant of arrest solely on the basis of the resolution of the prosecutor (in the instant case, the Office of the Special Prosecutor of the Ombudsman) who conducted the preliminary investigation, without having before him any of the evidence (such as complainants affidavit, respondents counter-affidavit, exhibits, etc.) which may have been submitted at the preliminary investigation?vi[7] In his separate petition, Rolando S. Narciso adopts the foregoing and raises no other distinct issue. Petitioners Ho and Narciso similarly contend that a judge, in personally determining the existence of probable cause, must have before him sufficient evidence submitted by the parties, other than the information filed by the investigating prosecutor, to support his conclusion and justify the issuance of an arrest warrant. Such evidence should not be merely described in a prosecutors resolution. Citing People vs. Inting,vii[8] petitioners insist that the judge must have before him the report, the affidavits, the transcripts of stenographic not es (if any), and all other supporting documents which are material in assisting the judge to make his determination. The Courts Ruling The petitions are meritorious. The pertinent provision of the Constitution reads: Section 2 *, Article III+. The right of the people to be secure in their persons, houses, papers, and effects against unreasonable searches and seizures of whatever nature and for any purpose shall be inviolable, and no search warrant or warrant of arrest shall issue except upon probable cause to be determined personally by the judge after examination under oath or affirmation of the complainant and the witnesses he may produce and particularly describing the place to be searched and the persons or things to be seized. (Underscoring supplied.) In explaining the object and import of the afore quoted constitutional mandate, particularly the power and the authority of judges to issue warrants of arrest, the Court elucidated in Soliven vs. Makasiarviii[9]: What the Constitution underscores is the exclusive and personal responsibility of the issuing judge to satisfy himself of the existence of probable cause. In satisfying himself of the existence of probable cause for the issuance of a warrant of arrest, the judge is not required to personally examine the complainant and his witnesses. Following established doctrine and procedure, he shall: (1) personally evaluate the report and the supporting documents submitted by the fiscal regarding the existence of probable cause and, on the basis thereof, issue a warrant of arrest; or (2) if on the basis thereof he finds no probable cause, he may disregard the fiscals report and require the submission of supporting affidavits of witnesses to aid him in arriving at a conclusion as to the existence of probable cause.ix[10] [underscoring supplied] We should stress that the 1987 Constitution requires the judge to determine probable cause personally. The word personally does not appear in the corresponding provisions of our previous Constitutions. This

emphasis shows the present Constitutions intent to place a greater degree of responsibility upon trial judges than that imposed under the previous Charters. While affirming Soliven, People vs. Intingx[11] elaborated on what determination of probable cause entails, differentiating the judges object or goal from that of the prosecutors. First, the determination of probable cause is a function of the Judge. It is not for the Provincial Fiscal or Prosecutor nor for the Election Supervisor to ascertain. Only the Judge and the Judge alone makes this determination. Second, the preliminary inquiry made by a Prosecutor does not bind the Judge. It merely assists him to make the determination of probable cause. The Judge does not have to follow what the Prosecutor presents to him. By itself, the Prosecutors certification of probable cause is ineffectual. It is the report, the affidavits the transcripts of stenographic notes (if any), and all other supporting documents behind the Prosecutors certification which are material in assisting the Judge to make his determination. And third, Judges and Prosecutors alike should distinguish the preliminary inquiry which determines probable cause for the issuance of a warrant of arrest from the preliminary investigation proper which ascertains whether the offender should be held for trial or released. Even if the two inquiries are conducted in the course of one and the same proceeding, there should be no confusion about the objectives. The determination of probable cause for the warrant of arrest is made by the Judge. The preliminary investigation proper -- whether or not there is reasonable ground to believe that the accused is guilty of the offense charged and, therefore, whether or not he should be subjected to the expense, rigors and embarrassment of trial -- is the function of the Prosecutor.xi[12] And clarifying the statement in People vs. Delgadoxii[13] -- that the trial court may rely on the resolution of the COMELEC to file the information, by the same token that it may rely on the certification made by the prosecutor who conducted the preliminary investigation, in the issuance of the warrant of arrest -- this Court underscored in Lim Sr. vs. Felixxiii[14] that *r+eliance on the COMELEC resolution or the Prosecutors certification presupposes that the records of either the COMELEC or the Prosecutor have been submitted to the Judge and he relies on the certification or resolution because the records of the investigation sustain the recommendation. We added, The warrant issues not on the strength of the certification standing alone but because of the records which sustain it. Summing up, the Court said : We reiterate the ruling in Soliven vs. Makasiar that the Judge does not have to personally examine the complainant and his witnesses. The Prosecutor can perform the same functions as a commissioner for the taking of the evidence. However, there should be a report and necessary documents supporting the Fiscals bare certification. All of these should be before the Judge. The extent of the Judges personal examination of the report and its annexes depends on the circumstances of each case. We cannot determine beforehand how cursory or exhaustive the Judges examination should be. The Judge has to exercise sound discretion for, after all, the personal determination is vested in the Judge by the Constitution. It can be as brief or as detailed as the circumstances of each case require. To be sure, the Judge must go beyond the Prosecutors certification and investigation report whenever necessary. He should call for *the+ complainant and *the+ witnesses themselves to answer the courts probing questions when the circumstances of the case so require.xiv[15] [underscoring supplied] The above rulings in Soliven, Inting and Lim Sr. were iterated in Allado vs. Dioknoxv[16] where we explained again what probable cause means. Probable cause for the issuance of a warrant of arrest is the existence of such

facts and circumstances that would lead a reasonably discreet and prudent person to believe that an offense has been committed by the person sought to be arrested.xvi[17] Hence, the judge, before issuing a warrant of arrest, must satisfy himself that based on the evidence submitted there is sufficient proof that a crime has been committed and that the person to be arrested is probably guilty thereof.xvii[18] At this stage of the criminal proceeding, the judge is not yet tasked to review in detail the evidence submitted during the preliminary investigation. It is sufficient that he personally evaluates such evidence in determining probable cause.xviii[19] In Webb vs. De Leon,xix[20] we stressed that the judge merely determines the probability, not the certainty, of guilt of the accused and, in doing so, he need not conduct a de novo hearing. He simply personally reviews the prosecutors initial determination finding probable cause to see if it is supported by substantial evidence. In the recent case of Roberts Jr. vs. Court of Appeals,xx[21] this Courts application of the dictum laid down in Soliven -- affirmed and fortified in Inting, Lim Sr., Allado and Webb -- should lay to rest the issue raised in the instant petitions. In Roberts Jr., this Court, through Mr. Justice Hilario G. Davide, Jr., set aside the order of the respondent judge directing inter alia the issuance of warrants of arrest against the accused, reasoning that said judge did not personally determine the existence of probable cause, since he had only the information, amended information, and Joint Resolution as bases thereof. He did not have the records or evidence supporting the prosecutors finding of probable cause.
Webb v. de Leon, 247 SCRA 652
FACTS: On June 19, 1994, the National Bureau of Investigation (NBI) filed with the Department of Justice a letter-complaint charging petitioners Hubert Webb, Michael Gatchalian, Antonio J. Lejano and six (6) other persons with the crime of Rape and Homicide of Carmela N. Vizconde, her mother Estrellita Nicolas-Vizconde, and her sister Anne Marie Jennifer in their home at Number 80 W. Vinzons, St., BF Homes Paranaque, Metro Manila on June 30, 1991. Forthwith, the Department of Justice formed a panel of prosecutors headed by Assistant Chief State Prosecutor Jovencio R. Zuno to conduct the preliminary investigation. ARGUMENTS: Petitioners fault the DOJ Panel for its finding of probable cause. They assail the credibility of Jessica Alfaro as inherently weak and uncorroborated due to the inconsistencies between her April 28, 1995 and May 22, 1995 sworn statements. They criticize the procedure followed by the DOJ Panel when it did not examine witnesses to clarify the alleged inconsistencies. Petitioners charge that respondent Judge Raul de Leon and, later, respondent Judge Amelita Tolentino issued warrants of arrest against them without conducting the required preliminary examination. Petitioners complain about the denial of their constitutional right to due process and violation of their right to an impartial investigation. They also assail the prejudicial publicity that attended their preliminary investigation. ISSUES: 1. Whether or not the DOJ Panel likewise gravely abused its discretion in holding that there is probable cause to charge them with the crime of rape and homicide 2. Whether or not respondent Judges de Leon and Tolentino gravely abused their discretion when they failed to conduct a preliminary examination before issuing warrants of arrest against them 3. Whether or not the DOJ Panel denied them their constitutional right to due process during their preliminary investigation 4. Whether or not the DOJ Panel unlawfully intruded into judicial prerogative when it failed to charge Jessica Alfaro in the information as an accused. HELD:

1. NO. 2. NO. 3. NO. There is no merit in this contention because petitioners were given all the opportunities to be heard. 4. NO. REASONS: 1. The Court ruled that the DOJ Panel did not gravely abuse its discretion when it found probable cause against the petitioners. A probable cause needs only to rest on evidence showing that more likely than not, a crime has been committed and was committed by the suspects. Probable cause need not be based on clear and convincing evidence of guilt, neither on evidence establishing guilt beyond reasonable doubt and definitely, not on evidence establishing absolute certainty of guilt. 2. The Court ruled that respondent judges did not gravely abuse their discretion. In arrest cases, there must be a probable cause that a crime has been committed and that the person to be arrested committed it. Section 6 of Rule 112 simply provides that upon filing of an information, the Regional Trial Court may issue a warrant for the accused. Clearly the, our laws repudiate the submission of petitioners that respondent judges should have conducted searching examination of witnesses before issuing warrants of arre st against them. 3. The DOJ Panel precisely ed the parties to adduce more evidence in their behalf and for the panel to study the evidence submitted more fully. 4. Petitioners argument lacks appeal for it lies on the faulty assumption that the decision whom to prosecute is a judicial function, the sole prerogative of the courts and beyond executive and legislative interference. In truth, the prosecution of crimes appertains to the executive department of government whose principal power and responsibility is to see that our laws are faithfully executed. A necessary component of this power is the right to prosecute their violators (See R.A. No. 6981 and section 9 of Rule 119 for legal basis). With regard to the inconsistencies of the sworn statements of Jessica Alfaro, the Court believes that these have been sufficiently explained and there is no showing that the inconsistencies were deliberately made to distort the truth. With regard to the petitioners complaint about the prejudicial publicity that attended their preliminary investigat ion, the Court finds nothing in the records that will prove that the tone and content of the publicity that attended the investigation of petitioners fatally infected the fairness and impartiality of the DOJ Panel. Petitioners cannot just rely on the subliminal effects of publicity on the sense of fairness of the DOJ Panel, for these are basically unbeknown and beyond knowing.

Roberts v. CA, 254 SCRA 307

I. THE FACTS Petitioners, who are corporate officers and members of the Board of Pepsi Cola Products Phils., Inc. were prosecuted in connection with the Pepsi Number Fever promotion by handlers of the supposedly winning 349 Pepsi crowns. Of the four cases filed against the petitioners, probable cause was found by the investigating prosecutor only for the crime of estafa, but not for the other alleged offenses. On 12 April 1993, the information was filed with the trial court without anything accompanying it. A copy of the investigating prosecutors Joint Resolution was forwarded to and received by the trial court only on 22 April 1993. However, no affidavits of the witnesses, transcripts of stenographic notes of the proceedings during the preliminary investigation, or other documents submitted in the course thereof were found in the records of the case as of 19 May 1993. On 15 April 1993, petitioners Roberts, et al. filed a petition for review to the Department of Justice seeking the reversal of the finding of probable cause by the investigating prosecutor. They also moved for the suspension of the proceedings and the holding in abeyance of the issuance of warrants of arrest against them. Meanwhile, the public prosecutor also moved to defer the arraignment of the accused-appellants pending the final disposition of the appeal to the Department of Justice.

On 17 May 1993, respondent Judge Asuncion issued the challenged order (1) denying, on the basis of Crespovs. Mogul, the foregoing motions respectively filed by the petitioners and the public prosecutor, and directing the issuance of the warrants of arrest after June 1993 and setting the arraignment on 28 June 1993. In part, respondent judge stated in his order that since the case is already pending in this Court for trial, following whatever opinion the Secretary of Justice may have on the matter would undermine the independence and integrity his court. To justify his order, he quoted the ruling of the Supreme Court in Crespo, which stated:
In order therefor to avoid such a situation whereby the opinion of the Secretary of Justice who reviewed the action of the fiscal may be disregarded by the trial court, the Secretary of Justice should, as far as practicable, refrain from entertaining a petition for review or appeal from the action of the fiscal, when the complaint or information has already been filed in Court. The matter should be left entirely for the determination of the Court.

Petitioners went to the Court of Appeals (CA), arguing that the respondent judge had not the slightest basis at all for determining probable cause when he ordered the issuance of warrants of arrest. After finding that a copy of the public prosecutors Joint Resolution had in fact been forwarded to, and received by, the trial court on 22 April 1993, the CA denied petitioners application for writ of preliminary injunction. The CA ruled that the Joint Resolution was sufficient in itself to have been relied upon by respondent Judge in convincing himself that probable cause indeed exists for the purpose of issuing the corresponding warrants of arrest and that the mere silence of the records or the absence of any express declaration in the questioned order as to the basis of such finding does not give rise to an adverse inference, for the respondent Judge enjoys in his favor the presumption of regularity in the performance of his official duty. Roberts, et al. sought reconsideration, but meanwhile, the DOJ affirmed the finding of probable cause by the investigating prosecutor. The CA therefore dismissed the petition for mootness. II. THE ISSUES 1. Did Judge Asuncion commit grave abuse of discretion in denying, on the basis of Crespo vs. Mogul, the motions to suspend proceedings and hold in abeyance the issuance of warrants of arrest and to defer arraignment until after the petition for review filed with the DOJ shall have been resolved? 2. Did Judge Asuncion commit grave abuse of discretion in ordering the issuance of warrants of arrest without examining the records of the preliminary investigation? 3. May the Supreme Court determine in this [sic] proceedings the existence of probable cause either for the issuance of warrants of arrest against the petitioners or for their prosecution for the crime of estafa? III. THE RULING [The Court, in a 7-5-2 vote, GRANTED the petition. It SET ASIDE the decision and resolution of the CA, the resolutions of the DOJ 349 Committee, and the order of respondent judge.] 1. YES, Judge Asuncion committed grave abuse of discretion in denying, on the basis of Crespo vs. Mogul, the motions to suspend proceedings and hold in abeyance the issuance of warrants of arrest and to defer arraignment until after the petition for review filed with the DOJ shall have been resolved. There is nothing in Crespo vs. Mogul which bars the DOJ from taking cognizance of an appeal, by way of a petition for review, by an accused in a criminal case from an unfavorable ruling of the investigating prosecutor. It merely advised the DOJ to, as far as practicable, refrain from entertaining a petition for review or appeal from the action of the fiscal, when the complaint or information has already been filed in Court.

Whether the DOJ would affirm or reverse the challenged Joint Resolution is still a matter of guesswork. Accordingly, it was premature for respondent Judge Asuncion to deny the motions to suspend proceedings and to defer arraignment on the following grounds:
This case is already pending in this Court for trial. To follow whatever opinion the Secretary of Justice may have on the matter would undermine the independence and integrity of this Court. This Court is still capable of administering justice.

The real and ultimate test of the independence and integrity of this court is not the filing of the aforementioned motions [to suspend proceedings and issuance of warrants of arrest and to defer arraignment] at that stage but the filing of a motion to dismiss or to withdraw the information on the basis of a resolution of the petition for review reversing the Joint Resolution of the investigating prosecutor. However, once a motion to dismiss or withdraw the information is filed the trial judge may grant or deny it, not out of subservience to the Secretary of Justice, but in faithful exercise of judicial prerogative. 2. YES, Judge Asuncion committed grave abuse of discretion in ordering the issuance of warrants of arrest without examining the records of the preliminary investigation. The teachings then of Soliven, Inting, Lim, Allado, and Webb reject the proposition that the investigating prosecutors certification in an information or his resolution which is made the basis for the filing of the information, or both, would suffice in the judicial determination of probable cause for the issuance of a warrant of arrest. In Webb, this Court assumed that since the respondent Judges had before them not only the 26-page resolution of the investigating panel but also the affidavits of the prosecution witnesses and even the counteraffidavits of the respondents, they (judges) made personal evaluation of the evidence attached to the records of the case. In this case, nothing accompanied the information upon its filing on 12 April 1993 with the trial court. A copy of the Joint Resolution was forwarded to, and received by, the trial court only on 22 April 1993. And as revealed by the certification of respondent judges clerk of court, no affidavits of the witnesses, transcripts of stenographic notes of the proceedings during the preliminary investigation, or other documents submitted in the course thereof were found in the records of this case as of 19 May 1993. Clearly, when respondent Judge Asuncion issued the assailed order of 17 May 1993 directing, among other things, the issuance of warrants of arrest, he had only the information, amended information, and Joint Resolution as bases thereof. He did not have the records or evidence supporting the prosecutor's finding of probable cause. And strangely enough, he made no specific finding of probable cause; he merely directed the issuance of warrants of arrest after June 21, 1993. It may, however, be argued that the directive presupposes a finding of probable cause. But then compliance with a constitutional requirement for the protection of individual liberty cannot be left to presupposition, conjecture, or even convincing logic. 3. NO, the Supreme Court MAY NOT determine in this [sic] proceedings the existence of probable cause either for the issuance of warrants of arrest against the petitioners or for their prosecution for the crime of estafa. Ordinarily, the determination of probable cause is not lodged with this Court. Its duty in an appropriate case is confined to the issue of whether the executive or judicial determination, as the case may be, of probable cause was done without or in excess of jurisdiction or with grave abuse of discretion amounting to want of jurisdiction. This is consistent with the general rule that criminal prosecutions may not be restrained or stayed by injunction, preliminary or final.

There are, however, exceptions to the foregoing rule. But the Court refused to reevaluate the evidence to determine if indeed there is probable cause for the issuance of warrants of arrest in this case. For the respondent judge did not, in fact, find that probable cause exists, and if he did he did not have the basis therefor. Moreover, the records of the preliminary investigation in this case are not with the Court. They were forwarded by the Office of the City Prosecutor of Quezon City to the DOJ in compliance with the latter's 1st Indorsement of 21 April 1993. The trial court and the DOJ must be required to perform their duty.

EXC: determination by administrative officials Morano v. Vivo, 20 SCRA 562

Facts: Chan Sau Wah, a Chinese citizen born in Fukien, China arrived in the Philippines on November 1961 to visit her cousin, Samuel Lee Malaps. She left China and her children by a first marriage: Fu Tse Haw and Fu Yan Kai both minors, in the care of neighbors in Fukien, China. Chan Sau wah arrived in the Philippines with Fu Yan Fun, her minor son also by the first marriage. Chan Sau Wah and her minor son Fu Yan Fun were permitted only into the Philippines under a temporary visitor's visa for two months and after they posted a cash bond of 4,000 pesos. On January 1962, Chan Sau Wah married Esteban Morano, a native-born Filipino citizen. Born to this union on September 1962 was Esteban Morano, Jr. To prolong their stay in the Philippines, Chan Sau Wah and Fu Yan Fun obtained several extensions. The last extension expired on September 10, 1962. In a letter dated August 31, 1962, the Commissioner of Immigration ordered Chan Sau Wah and her son, Fu Yan Fun, to leave the country on or before September 10, 1962 with a warning that upon failure so to do, he will issue a warrant for their arrest and will cause the confiscation of their bond. Issue: Whether or Not the issuance of the warrant of arrest is unconstitutional. Held: Chan Sau Wah entered the Philippines on a tourist-temporary visitor's visa. She is a non-immigrant. Under Section 13 just quoted, she may therefore be admitted if she were a qualified and desirable alien and subject to the provisions of the last paragraph of Section 9. Therefore, first, she must depart voluntarily to some foreign country; second, she must procure from the appropriate consul the proper visa; and third, she must thereafter undergo examination by the officials of the Bureau of Immigration at the port of entry for determination of her admissibility in accordance with the requirements of the immigration Act. This Court in a number of cases has ruled, and consistently too, that an alien admitted as a temporary visitor cannot change his or her status without first departing from the country and complying with the requirements of Section 9 of the Immigration Act. The gravamen of petitioners' argument is that Chan Sau Wah has, since her entry, married in Manila a native-born Filipino, Esteban Morano. It will not particularly help analysis for petitioners to appeal to family solidarity in an effort to thwart her deportation. Chan Sau Wah, seemingly is not one who has a high regard for such solidarity. Proof: She left two of her children by the first marriage, both minors, in the care of neighbors in Fukien, China.Then, the wording of the statute heretofore adverted to is a forbidding obstacle which will prevent this Court from writing into the law an additional provision that marriage of a temporary alien visitor to a Filipino would ipso facto make her a permanent resident in his country. This is a field closed to judicial action. No breadth of discretion is allowed. We cannot insulate her from the State's power of deportation. it would be an easy matter for an alien woman to enter the Philippines as a temporary visitor, go through a mock marriage, but actually live with another man as husband and wife, and thereby skirt the provisions of our immigration law. Also, a woman of undesirable character may enter this country, ply a pernicious trade, marry a Filipino, and again throw overboard Sections 9 and 13 of the Act. Such a flanking movement, we are confident, is impermissible.Recently we confirmed the rule that an alien wife of a Filipino may not stay permanently without first departing from the Philippines. Reason: Discourage entry under false pretenses.

Harvey v. Santiago, 162 SCRA 840

Facts: This is a petition for Habeas Corpus. Petitioners are the following: American
nationals Andrew Harvey, 52 and Jonh Sherman 72. Dutch Citizen Adriaan Van Den Elshout, 58. All reside at Pagsanjan Laguna respondent Commissioner Miriam Defensor Santiago issued Mission Orders to the Commission of Immigration and Deportation (CID) to apprehended petitioners at their residences. The Operation Report read that Andrew Harvey was found together with two young boys. Richard Sherman was found with two naked boys inside his room. While Van Den Elshout in the after Mission Report read that two children of ages 14 and 16 has been under his care and subjects confirmed being live-in for sometime now. Seized during the petitioners apprehension were rolls of photo negatives and photos of suspected child prostitutes shown in scandalous poses as well as boys and girls engaged in sex. Posters and other literature advertising the child prostitutes were also found. Petitioners were among the 22 suspected alien pedophiles. They were apprehended 17 February1988 after close surveillance for 3 month of the CID in Pagsanjan, Laguna. 17 of the arrested aliens opted for self-deportation. One released for lack of evidence, another charged not for pedophile but working with NO VISA, the 3 petitioners chose to face deportation proceedings. On 4 March1988, deportation proceedings were instituted against aliens for being undesirable aliens under Sec.69 of Revised Administrative Code. Warrants of Arrest were issued 7March1988 against petitioners for violation of Sec37, 45 and 46 of Immigration Act and sec69 of Revised Administrative Code. Trial by the Board of Special Inquiry III commenced the same date. Petition for bail was filed 11March 1988 but was not granted by the Commissioner of Immigration. 4 April1988 Petitioners filed a petition for Writ of Habeas Corpus. The court heard the case on oral argument on 20 April 1988.

Issues:
(1) Whether or Not the Commissioner has the power to arrest and detain petitioners pending determination of existence of probable cause. (2) Whether or Not there was unreasonable searches and seizures by CID agents.

(3) Whether or Not the writ of Habeas Corpus may be granted to petitioners.

Held: While pedophilia is not a crime under the Revised Penal Code, it violates the
declared policy of the state to promote and protect the physical, moral, spiritual and social well being of the youth. The arrest of petitioners was based on the probable cause determined after close surveillance of 3 months. The existence of probable cause justified the arrest and seizure of articles linked to the offense. The articles were seized as an incident to a lawful arrest; therefore the articles are admissible evidences (Rule 126, Section12 of Rules on Criminal Procedure). The rule that search and seizures must be supported by a valid warrant of arrest is not an absolute rule. There are at least three exceptions to this rule. 1.) Search is incidental to the arrest. 2.) Search in a moving vehicle. 3.) Seizure of evidence in plain view. In view of the foregoing, the search done was incidental to the arrest. The filing of the petitioners for bail is considered as a waiver of any irregularity attending their arrest and estops them from questioning its validity. Furthermore, the deportation charges and the hearing presently conducted by the Board of Special Inquiry made their detention legal. It is a fundamental rule that habeas corpus will not be granted when confinement is or has become legal, although such confinement was illegal at the beginning. The deportation charges instituted by the Commissioner of Immigration are in accordance with Sec37 (a) of the Philippine Immigration Act of 1940 in relation to sec69 of the Revised Administrative code. Section 37 (a) provides that aliens shall be arrested and deported upon warrant of the Commissioner of Immigration and Deportation after a determination by the Board of Commissioners of the existence of a ground for deportation against them. Deportation proceedings are administrative in character and never construed as a punishment but a preventive measure. Therefore, it need not be conducted strictly in accordance with ordinary Court proceedings. What is essential is that there should be a specific charge against the alien intended to be arrested and deported. A fair hearing must also be conducted with assistance of a counsel if desired. Lastly, the power to deport aliens is an act of the State and done under the authority of the sovereign power. It a police measure against the undesirable aliens whose continued presence in the country is found to be injurious to the public good and tranquility of the people.

EXC to EXC Salazar v. Achacoso, 183 SCRA 145

F: Pursuant to the powers vested by PD 1920 and EO 1022, POEA Administrator Achacoso ordered the closure of the recruitment agency of Horty Salazar, having verified that she had no license to operate a recruitment agency. He further ordered the seizure of the documents and paraphernalias, being used or intended to be used as the means of commiting illegal recruitment. This order was enforced on 26 January 1988. Petitioner filed this suit for prohibition. Issue: May the POEA (or the Sec. of Labor) validly issue warrants of serach and seizure (or arrest ) under Art. 38 of the Labor Code? HELD: NO. The provisions of PD 1920 and EO 1022, now embodied in Art. 38 of the Labor Code, are the dying vestiges of authoritarian rule in its twilights moments. Under Art. III, Sec 2 of the 1987 Constitution, it is only judges and no other, who may issue warrants of arrest and search. The exception is in cases of deportation of illegal and undesirable aliens, whom the President of the Commissioner of Immigration may order arrested, following a final order of deportation, for the purpose of deportation. The Sec. of Labor , not being a judge. may no longer issue search or arrest warrants. Hence, the authorities must go through the judicial process. To that extent, we declare Art. 38, par. C of the Labor Code, unconstitutional and of no force and effect. a. Existence of probable cause. Probable cause is such facts and circumstances as would reasonably make a prudent man believe that a crime have been committed and that the documents or things sought to be searched and seized are in the possession of the person against whom the warrant is sought. Without probable cause, there can be no valid search warrant. See Pasion vda. de Garcia v. Locsin, 65 Phil. 689, (1938)

Source: http://www.shvoong.com/law-and-politics/1767268-case-digest-salazar-achacoso-183/#ixzz2I7CV1qgK

Examination of Applicant by the judge Alvarez v. CFI, 64 Phil 33

FACTS: On June 3 1936, Judge Eduardo Gutierrez David of theCourt of First Instance of Tayabas issued a search warrant onthe basis of affidavit of Agent Mariano Almeda in whose oathhe declared that he had no personal knowledge but throughinformation from a reliable source. In other words, theapplicant's knowledge of facts is based on a mere hearsay.In the affidavit presented to the judge, the description isas follows:"That there are being kept is said premises booksdocuments, receipts, lists chits, and other papers used byhim in connection with his activities as money lender,charging a usurious rate of interests, in violation of the law."At 7 pm on June 4, by virtue of the warrant, severalagents of the Anti-Usury Board enterd the store andresidence of Narciso Alvarez seized some articles such asinternal revenue license, ledger, journals. cash bonds, checkstubs, memorandums, blackboards, contracts, inventories,bill of lading, credit receipts, correspondence, receipt books,promissory notes and checks.On July 8, Alvarez filed a petition alleging that the searchwas illegal based on the lack of personal knowledge, that itwas made at night and for non compliance in theparticularity description rule in issuing warrant.On September 10, the Court of First Instance ruledagainst the Alvarez and upheld the validity of the searchwarrant.

ISSUES: (1) W/N the search warrant is legal when the affidavit isbased on hearsay.(2) W/N a search warrant can be made at night.(3) W/N the search warrant satisfies the particularity of description as required by the law. RULING: (1) No. The search warrant is ILLEGAL because the affidavitis based on mere hearsay.RATIO: The general rule is that when the affidavit of theapplicant or complainant contains sufficient facts within hispersonal and direct knowledge, it is sufficient if the judge issatisfied that there exists probable cause.But when the applicant's knowledge of the facts is merehearsay, the affidavit of one or more witnesses havingpersonal knowledge of facts is necessary. The Court heldthat the warrant is illegal because it is based on the affidavitof an agent who had no personal knowledge of the facts. The true test of sufficiency of a deposition or affidavit towarrant issuance of a search warrant is whether it has beendrawn in such a manner that perjury could be chargedthereon and affiant be held liable for damages caused.(2) Yes, the search can be made at night.RATIO: Section 101 of General Orders number 58 authorizsa search made at night when it is positively asserted thatthe property is on the person or in the place ordered tobe searched. However, since the search warrant is declaredillegal (RULING 1), such search could not be legally made atnight.(3) Yes, it satisfied the requirement of particularity of description. RATIO: Article III of the Constitution and section 97 of General Orders Number 58 requires that the affidavit mustcontain a particular description of the placed to be searchedand the person or thing to be seized.But, where, by the nature of the goods to be seized, theirdescription must be rather general, it is not required thattechnical description be given, as this would mean that nowarrant could issue.Based on the description of the affidavit, and taking intoconsideration the nature of the articles as described it isclear that no other more adequate and detailed descriptioncould have been given, particularly because it is difficult togive a particular description of the contents thereof. The description so made substantially complies with thelegal provisions because the officer of the law who executedthe warrant was thereby placed in a position enabiling himto identify the articles in question, which he did.
Burgos v. Chief of Staff, supra Mata v. Bayona, 128 SCRA 388 Facts: Soriano Mata was accused under Presidential Decree (PD) 810, as amended by PD 1306, the information against him alleging that Soriano Mata offered, took and arranged bets on the Jai Alai game by selling illegal tickets known as Masiao tickets without any authority from the Philippine Jai Alai & Amusement Corporation or from the government authorities concerned. Mata claimed that during the hearing of the case, he discovered that nowhere from the records of the said case could be found the search warrant and other pertinent papers connected to the issuance of the same, so that he had to inquire from the City Fiscal its whereabouts, and to which inquiry Judge Josephine K. Bayona, presiding Jufe of the City Court of Ormoc replied, it is with the court. The Judge then handed the records to the Fiscal who attached them to the records. This led Mata to file a motion to quash and annul the search warrant and for the return of the articles seized, citing and invoking, among others, Section 4 of Rule 126 of the Revised Rules of Court. The motion was denied by the Judge on 1 March 1979, stating that the court has made a thorough investigation and examination under oath of Bernardo U. Goles and Reynaldo T. Mayote, members of the Intelligence Section of 352nd PC Co./Police District II INP; that in fact the court made a certification to that effect; and that the fact that documents relating to the search warrant were not attached immediately to the record of the criminal case is of no moment, considering that the rule does not specify when these documents are to be attached to the records. Matas motion for reconsideration of the

aforesaid order having been denied, he came to the Supreme Court, with the petition for certiorari, praying, among others, that the Court declare the search warrant to be invalid for its alleged failure to comply with the requisites of the Constitution and the Rules of Court, and that all the articles confiscated under such warrant as inadmissible as evidence in the case, or in any proceedings on the matter. Issue: Whether the judge must before issuing the warrant personally examine on oath or affirmation the complainant and any witnesses he may produce and take their depositions in writing, and attach them to the record, in addition to any affidavits presented to him. Held: Under the Constitution no search warrant shall issue but upon probable cause to be determined by the Judge or such other responsible officer as may be authorized by law after examination under oath or affirmation of the complainant and the witnesses he may produce. More emphatic and detailed is the implementing rule of the constitutional injunction, The Rules provide that the judge must before issuing the warrant personally examine on oath or affirmation the complainant and any witnesses he may produce and take their depositions in writing, and attach them to the record, in addition to any affidavits presented to him. Mere affidavits of the complainant and his witnesses are thus not sufficient. The examining Judge has to take depositions in writing of the complainant and the witnesses he may produce and to attach them to the record. Such written deposition is necessary in order that the Judge may be able to properly determine the existence or nonexistence of the probable cause, to hold liable for perjury the person giving it if it will be found later that his declarations are false. We, therefore, hold that the search warrant is tainted with illegality by the failure of the Judge to conform with the essential requisites of taking the depositions in writing and attaching them to the record, rendering the search warrant invalid. Ponsica v. Ignalaga, 152 SCRA 647 Particularity of Description Stonehill v. Diokno, supra Alvarez v. CFI, 64 SCRA 33 Burgos v. Chief of Staff, supra Nolasco v. Pano, 139 SCRA 152

FACTS: Aquilar-Roque and Nolasco were arrested by a Constabulary Security Group (CSG) at the intersection of Mayon Street, Quezon City On the same day, a searched was conducted. Ct. Col. Virgilio Saldajeno; applied for search warrant from the respondent Hon. Ernani Cruz Pano, after a month of round the clock surveillance of the premises as a suspected underground house of the CPP/NPA, particularly connected to MV Karagatan / Pena Andrea cases. The searching party seized 428 documents and written materials, and additionally a portable typewriter and 2 wooden boxes. The City Fiscal information for violation of PD No. 33, Illegal Possession of Subversive Documents Petitioners contend that the Search Warrant is void because it is a general warrant since it does not sufficiently describe with particularly the things subject of the search and seizure and that probable cause had not been properly established for lack of searching questions propounded to the applicants witness. Disputed Search Warrant: Documents, papers and other records of the communist party of the Philippines / New Peoples Army and or the National Democratic Front, such as Minutes of the Party Philippines.. ISSUE: Whether or not the search warrant issued was of general warrant and illegal?

HELD: The search warrant is of General, thus, it was hereby annulled by set aside. RATIONALE: The Search Warrant does not specify what the subversive books and instructions are; what are the manuals not otherwise available to the public certain to make them subversive or to enable them to be used for the crime of rebellion. There is absent a definite guideline to the searching team as to what items might be lawfully seized thus giving the officers of the law discretion regarding what articles they should seize as, in fact, taken also were a portable typewriter. Mere generalization will not suffice and odes not satisfy the requirements of probable cause upon which a warrant may issue. Facts: The present case was subject for resolution. Supreme Court held in a criminal case that the arrest of the petitioners was illegal, annulling the decision of respondent Judge Pao, and that the seizure of the items by virtue of the warrant by the same respondent judge are inadmissible as evidence in the Subversive Documents case. However the Court held that the items were to be retained in case it would be used as evidence in a separate criminal case pending before the Special Military Commission No.1, returning the rest which are determined irrelevant by petitioner. Petitioners questioned the portion of the decision regarding the retention of the properties seized. One of the petitioners also assailed the respondents claim that the search was incidental to her arrest for the crime of rebellion. Issue: Whether or not some of the properties seized may be introduced as evidence in a separate criminal case.
20th Century Fox v. CA, 164 SCRA 655

I.

THE FACTS

Petitioner 20th Century Fox Film Corporation sought the assistance of the NBI in conducting searches and seizures in connection with the NBIs anti-film piracy campaign. Petitioner alleged that certain videotape outlets all over Metro Manila are engaged in the unauthorized sale and renting out of copyrighted films in violation of PD No. 49 (the old Intellectual Property Law). The NBI conducted surveillance and investigation of the outlets pinpointed by the petitioner and subsequently filed three (3) applications for search warrants against the video outlets owned by the private respondents. The lower court issued the desired search warrants. The NBI, accompanied by the petitioner's agents, raided the video outlets and seized the items described in the three warrants. Private respondents later filed a motion to lift the search warrants and release the seized properties, which was granted by the lower court. Petitioners motion for reconsideration was denied by the lower court. The CA affirmed the trial court. II. THE ISSUE

Did the judge properly lift the search warrants he issued earlier? III. THE RULING [The Court DISMISSED the petition and AFFIRMED the questioned decision and resolution of the CA.] YES, the judge properly lifted the search warrants he issued earlier. The lower court lifted the three (3) questioned search warrants in the absence of probable cause that the private respondents violated P.D. 49. NBI agents who acted as witnesses during the application for search warrant did not have personal knowledge of the subject matter of their testimony, which was the alleged commission of the offense of piracy by the private respondents. Only the petitioners counsel who was also a witness during the application stated that he had personal knowledge that the confiscated tapes owned by the private respondents were pirated tapes taken from master tapes belonging to the petitioner. The lower court lifted the warrants, declaring that the testimony of petitioners counsel did not have much credence because the master tapes of the allegedly pirated tapes were not shown to the court during the application. The presentation of the master tapes of the copyrighted films, from which the pirated films were allegedly copied, was necessary for the validity of search warrants against those who have in their possession the pirated films. The petitioner's argument to the effect that the presentation of the master tapes at the time of application may not be necessary as these would be merely evidentiary in nature and not determinative of whether or not a probable cause exists to justify the issuance of the search warrants is not meritorious. The court cannot presume that duplicate or copied tapes were necessarily reproduced from master tapes that it owns. The essence of a copyright infringement is the similarity or at least substantial similarity of the purported pirated works to the copyrighted work. Hence, the applicant must present to the court the copyrighted films to compare them with the purchased evidence of the video tapes allegedly pirated to determine whether the latter is an unauthorized reproduction of the former. This linkage of the copyrighted films to the pirated films must be established to satisfy the requirements of probable cause. Mere allegations as to the existence of the copyrighted films cannot serve as basis for the issuance of a search warrant.
Columbia Pictures v. Flores, 223 SCRA 761

I. THE FACTS As a consequence of a complaint filed by the Motion Picture Association of America, Inc., NBI agents conducted surveillance operations on certain video establishments, among them respondent FGT Video Network, Inc. (FGT), for unauthorized sale, rental, reproduction and/or disposition of copyrighted film," a violation of PD 49 (the old Intellectual Property Law). After an NBI agent was able to have copyrighted motion pictures Cleopatra (owned by 20th Century Fox) and The Ten Commandments (owned by Paramount) reproduced in video format in FGT, the NBI applied for and was able to obtain from the respondent judge the subject Search Warrant No. 45 which reads:
TO ANY PEACE OFFICER: GREETINGS:

It appearing to the satisfaction of the Undersigned after examining under oath NBI Senior Agent Lauro C. Reyes and his witnesses Mr. Danilo Manalang and Ms. Rebecca Benitez-Cruz, that there is a probable cause to believe that Violation of Section 56 P.D. No. 49 as amended by P.D. No. 1988 (otherwise known as the Decree on Protection of Intellectual Property) has been committed and that there are good and sufficient reasons to believe that FGT Video Network, Inc., Manuel Mendoza, Alfredo C. Ongyanco, Eric Apolonio, Susan Yang and Eduardo Yotoko are responsible and have in control/possession at No. 4 Epifanio de los Santos corner Connecticut, Greenhills, San Juan, Metro Manila (per attached sketch and list of MPAA member Company Titles) the following properties to wit: (a) Pirated video tapes of the copyrighted motion pictures/films the titles of which are mentioned in the attached list; (b) Posters, advertising leaflets, flyers, brochures, invoices, lists of titles being reproduced or retaped, journals, ledgers, jon ( sic) order slips, delivery slips and books of accounts bearing and/or mentioning the pirated films with titles (as per attached list), or otherwise used in the reproduction/retaping business of the defendants; (c) Television sets, video cassette recorders, rewinders, tape head cleaners, accessories, equipment and other machines and paraphernalia or materials used or intended to be used in the unlawful sale, lease, distribution, or possession for purpose of sale, lease, distribution, circulation or public exhibition of the above-mentioned pirated video tapes which they are keeping and concealing in the premises above-described, which should be seized and brought to the Undersigned. You are hereby commanded to make an immediate search at any time in the day between 8:00 A.M. to 5:00 P.M. of the premises above-described and forthwith seize and take possession of the above-enumerated personal properties, and bring said properties to the undersigned immediately upon implementation to be dealt with as the law directs.

In the course of the implementation of the search warrant in the premises of FGT, the NBI agents found and seized various video tapes of copyrighted films owned and exclusively distributed by petitioners. Also seized were machines and equipment, television sets, paraphernalia, materials, accessories, rewinders, tape head cleaners, statements of order, return slips, video prints, flyers, production orders, and posters. FGT moved for the release of the seized television sets, video cassette recorders, rewinders, tape head cleaners, accessories, equipment and other machines or paraphernalia seized by virtue of the subject warrant. It argued that as a licensed video reproducer, it had the right possess the seized reproduction equipment, which are not illegal per se, but are rather exclusively used and intended to be used for reproduction and not in the sale, lease, distribution or possession for purposes of sale, lease distribution, circulation or public exhibition of pirated video tapes. Finding that FGT was a registered and duly licensed distributor and in certain instances and under special instructions and conditions reproducer of videograms and that, therefore, its right to possess and use the seized equipment had been placed in serious doubt, the lower court ordered the return of the television sets, video cassette recorders, rewinders, tape head cleaners, accessories, equipment and other machines or paraphernalia to FGT.

II. THE ISSUE Did the respondent judge act with grave abuse of discretion amounting to lack of jurisdiction in ordering the immediate return of some of the items seized by virtue of the search warrant?

III. THE RULING [The High Tribunal DISMISSED the petition and AFFIRMED the order of the respondent Judge Flores.] NO, the respondent judge DID NOT act with grave abuse of discretion amounting to lack of jurisdiction in ordering the immediate return of some of the items seized by virtue of the search warrant.

Search Warrant No. 45 fails to satisfy the test of legality. This is more so because the Court has previously decided a case dealing with virtually the same kind of search warrant. In 20th Century Fox vs. CA, the Court upheld the legality of the order of the lower court lifting the search warrant issued under circumstances similar to those obtaining in the case at bar. A striking similarity between this case and 20th Century Fox is the fact that Search Warrant No. 45, specifically paragraph (c) thereof describing the articles to be seized, contains an almost identical description as the warrant issued in the 20th Century Fox case, to wit:
(c) Television sets, Video Cassettes Recorders, rewinders, tape head cleaners, accessories, equipments and other machines used or intended to be used in the unlawful reproduction, sale, rental/lease, distribution of the above-mentioned video tapes which she is keeping and concealing in the premises above-described.

On the propriety of the seizure of the articles above-described, the Court held in 20th Century Fox:
Television sets, video cassette recorders, rewinders and tape cleaners are articles which can be found in a video tape store engaged in the legitimate business of lending or renting out betamax tapes. In short, these articles and appliances are generally connected with, or related to a legitimate business not necessarily involving piracy of intellectual property or infringement of copyright laws. Hence, including these articles without specification and/or particularity that they were really instruments in violating an Anti-Piracy law makes the search warrant too general which could result in the confiscation of all items found in any video store.

The language used in paragraph (c) of Search Warrant No. 45 is thus too all-embracing as to include all the paraphernalia of FGT in the operation of its business. As the search warrant is in the nature of a general one, it is constitutionally objectionable. The Court concluded that the respondent judge did not gravely abuse his discretion in ordering the immediate release of the enumerated items, but that he was merely correcting his own erroneous conclusions in issuing Search Warrant No. 45. This can be gleaned from his statement that . . . the machines and equipment could have been used or intended to be used in the illegal reproduction of tapes of the copyrighted motion pictures/films, yet, it cannot be said with moral certainty that the machines or equipment(s) were used in violating the law by the mere fact that pirated video tapes of the copyrighted motion pictures/films were reproduced. As already stated, FGT Video Network, Inc. is a registered and duly licensed distributor and in certain instances and under special instructions . . . reproducer of videograms, and as such, it has the right to keep in its possession, maintain and operate reproduction equipment(s) and paraphernalia(s) . http://philippineconstitutionallawdigests.blogspot.com/2012/03/20th-century-fox-film-vs-court-of.html
Pp v. Tee, G.R. No. 140546-47, January 20, 2003

FACTS: Appellant is a Chinese national in his forties, a businessman, and a resident of Baguio City. A raid conducted by operatives of the National Bureau of Investigation (NBI) and Philippine National Police Narcotics Command (PNP NARCOM) at premises allegedly leased by appellant and at his residence yielded huge quantities of marijuana. Appellant moved to quash the search warrant on the ground that it was too general and that the NBI had not complied with the requirements for the issuance of a valid search warrant. The pendency of said motion, however, did not stop the filing of the appropriate charges against appellant. In an information dated July 24, 1998, the City Prosecutor of Baguio City charged Modesto Tee, alias Estoy Tee, with illegal possession of marijuana. ISSUE: Whether or not the appellant's contention that the description on the serach warrant which says an undetermined amount of marijuana, was too general and hence makes the warrant void for vagueness.

HELD: SC held that the appellants contention, has no leg to stand on. The constitutional requirement of reasonable particularity of description of the things to be seized is primarily meant to enable the law enforcers serving the warrant to: (1) readily identify the properties to be seized and thus prevent them from seizing the wrong items; and (2) leave said peace officers with no discretion regarding the articles to be seized and thus prevent unreasonable searches and seizures. What the Constitution seeks to avoid are search warrants of broad or general characterization or sweeping descriptions, which will authorize police officers to undertake a fishing expedition to seize and confiscate any and all kinds of evidence or articles relating to an offense. However, it is not required that technical precision of description be required, particularly, where by the nature of the goods to be seized, their description must be rather general, since the requirement of a technical description would mean that no warrant could issue.

Microsoft Corp v. Maxicorp, Inc., G.R. No. 140946, Sept 13, 2004

On 25 July 1996, National Bureau of Investigation (NBI) Agent Dominador Samiano, Jr. (NBI Agent Samiano) filed several applications for search warrants in the RTC against Maxicorp for alleged violation of Section 29 of PD 49 and Article 189 of the RPC. After conducting a preliminary examination of the applicant and his witnesses, Judge William M. Bayhon issued Search Warrants Nos. 96-451, 96-452, 96-453 and 96-454, all dated 25 July 1996, against Maxicorp. Armed with the search warrants, NBI agents conducted on 25 July 1996 a search of Maxicorps premises and seized property fitting the description stated in the search warrants. On 2 September 1996, Maxicorp filed a motion to quash the search warrants alleging that there was no probable cause for their issuance and that the warrants are in the form of general warrants. The RTC denied Maxicorps motion on 22 January 1997. The RTC also denied Maxicorps motion for reconsideration. The RTC found probable cause to issue the search warrants after examining NBI Agent Samiano, John Benedict Sacriz (Sacriz), and computer technician Felixberto Pante (Pante). The three testified on what they discovered during their respective visits to Maxicorp. NBI Agent Samiano also presented certifications from petitioners that they have not authorized Maxicorp to perform the witnessed activities using petitioners products. On 24 July 1997, Maxicorp filed a petition for certiorari with the Court of Appeals seeking to set aside the RTCs order. On 23 December 1998, the Court of Appeals reversed the RTCs order denying Maxicorps motion to quash the search warrants. Petitioners moved for reconsideration. The Court of Appeals denied petitioners motion on 29 November 1999. The Court of Appeals held that NBI Agent Samiano failed to present during the preliminary examination conclusive evidence that Maxicorp produced or sold the counterfeit products. The Court of Appeals pointed out that the sales receipt NBI Agent Samiano presented as evidence that he bought the products from Maxicorp was in the name of a certain Joel Diaz. Hence, this petition. WHETHER THE SEARCH WARRANTS ARE GENERAL WARRANTS.

A search warrant must state particularly the place to be searched and the objects to be seized. The evident purpose for this requirement is to limit the articles to be seized only to those particularly described in the search warrant. This is a protection against potential abuse. It is necessary to leave the officers of the law with no discretion regarding what articles they shall seize, to the end that no unreasonable searches and seizures be committed.[33] In addition, under Section 4, Rule 126 of the Rules of Criminal Procedure, a search warrant shall issue in connection with one specific offense. The articles described must bear a direct relation to the offense for which the warrant is issued.[34] Thus, this rule requires that the warrant must state that the articles subject of the search and seizure are used or intended for use in the commission of a specific offense. Maxicorp argues that the warrants issued against it are too broad in scope and lack the specificity required with respect to the objects to be seized. After examining the wording of the warrants issued, the Court of Appeals ruled in favor of Maxicorp and reversed the RTCs Order thus: Under the foregoing language, almost any item in the petitioners store can be seized on the ground that it is used or intended to be used in the illegal or unauthorized copying or reproduction of the private respondents software and their manuals.[35] The Court of Appeals based its reversal on its perceived infirmity of paragraph (e) of the search warrants the RTC issued. The appellate court found that similarly worded warrants, all of which noticeably employ the phrase used or intended to be used, were previously held void by this Court.[36] The disputed text of the search warrants in this case states: a) Complete or partially complete reproductions or copies of Microsoft software bearing the Microsoft copyrights and/or trademarks owned by MICROSOFT CORPORATION contained in CD-ROMs, diskettes and hard disks; Complete or partially complete reproductions or copies of Microsoft instruction manuals and/or literature bearing the Microsoft copyrights and/or trademarks owned by MICROSOFT CORPORATION; Sundry items such as labels, boxes, prints, packages, wrappers, receptacles, advertisements and other paraphernalia bearing the copyrights and/or trademarks owned by MICROSOFT CORPORATION; Sales invoices, delivery receipts, official receipts, ledgers, journals, purchase orders and all other books of accounts and documents used in the recording of the reproduction and/or assembly, distribution and sales, and other transactions in connection with fake or counterfeit products bearing the Microsoft copyrights and/or trademarks owned by MICROSOFT CORPORATION; Computer hardware, including central processing units including hard disks, CD-ROM drives, keyboards, monitor screens and diskettes, photocopying machines and other equipment or paraphernalia used or intended to be used in the illegal and unauthorized copying or reproduction of Microsoft software and their manuals, or which contain, display or otherwise exhibit, without the authority of MICROSOFT CORPORATION, any and all Microsoft trademarks and copyrights; and Documents relating to any passwords or protocols in order to access all computer hard drives, data bases and other information storage devices containing unauthorized Microsoft software. [37] (Emphasis supplied)

b)

c)

d)

e)

f)

It is only required that a search warrant be specific as far as the circumstances will ordinarily allow. [38] The description of the property to be seized need not be technically accurate or precise. The nature of the description should vary according to whether the identity of the property or its character is a matter of concern.[39] Measured against this standard we find that paragraph (e) is not a general warrant. The articles to be seized were not only sufficiently identified physically, they were also specifically identified by stating their relation to the offense charged. Paragraph (e) specifically refers to those articles used or intended for use in the illegal and unauthorized copying of petitioners software. This language meets the test of specificity.[40] The cases cited by the Court of Appeals are inapplicable. In those cases, the Court found the warrants too broad because of particular circumstances, not because of the mere use of the phrase used or intended to be used. In Columbia Pictures, Inc. v. Flores, the warrants ordering the seizure of television sets, video cassette recorders, rewinders and tape cleaners x x x were found too broad since the defendant there was a licensed distributor of video tapes.[41] The mere presence of counterfeit video tapes in the defendants store does not mean that the machines were used to produce the counterfeit tapes. The situation in this case is different. Maxicorp is not a licensed distributor of petitioners. In Bache & Co. (Phil.), Inc., et al. v. Judge Ruiz, et al., the Court voided the warrants because they authorized the seizure of records pertaining to all business transactions of the defendant.[42] And in 20th Century Fox Film Corp. v. Court of Appeals, the Court quashed the warrant because it merely gave a list of articles to be seized, aggravated by the fact that such appliances are generally connected with the legitimate business of renting out betamax tapes. [43] However, we find paragraph (c) of the search warrants lacking in particularity. Paragraph (c) states: c) Sundry items such as labels, boxes, prints, packages, wrappers, receptacles, advertisements and other paraphernalia bearing the copyrights and/or trademarks owned by MICROSOFT CORPORATION; The scope of this description is all-embracing since it covers property used for personal or other purposes not related to copyright infringement or unfair competition. Moreover, the description covers property that Maxicorp may have bought legitimately from Microsoft or its licensed distributors. Paragraph (c) simply calls for the seizure of all items bearing the Microsoft logo, whether legitimately possessed or not. Neither does it limit the seizure to products used in copyright infringement or unfair competition. Still, no provision of law exists which requires that a warrant, partially defective in specifying some items sought to be seized yet particular with respect to the other items, should be nullified as a whole. A partially defective warrant remains valid as to the items specifically described in the warrant. [44] A search warrant is severable, the items not sufficiently described may be cut off without destroying the whole warrant.[45] The exclusionary rule found in Section 3(2) of Article III of the Constitution renders inadmissible in any proceeding all evidence obtained through unreasonable searches and seizure. Thus, all items seized under paragraph (c) of the search warrants, not falling under paragraphs a, b, d, e or f, should be returned to Maxicorp. WHEREFORE, we PARTIALLY GRANT the instant petition. The Decision of the Court of Appeals dated 23 December 1998 and its Resolution dated 29 November 1999 in CA-G.R. SP No. 44777 are REVERSED and SET ASIDE except with respect to articles seized under paragraph (c) of Search Warrants Nos. 96-451, 96-452, 96453 and 96-454. All articles seized under paragraph (c) of the search warrants, not falling under paragraphs a, b, d, e or f, are ordered returned to Maxicorp, Inc. immediately.

In connection with one particular offense

Prudente v. Dayrit, G.R. No. 82870 December 14, 1989, 180 SCRA 69

4. Other forms of search Drug testing as search SJS v. Dangerous Drugs Board, G.R. No. 157870, November 3, 2008 5. Properties Subject to Seizure (See Sec. 3 Rule 126, Revised Rules of Criminal Procedure) Pp v. Marcos, 117 SCRA 999

Uy Khetin vs. Villareal, 42 Phil 886 Facts: -On April 30, 1919, one Ramon Gayanilo, corporal of the Philippine Constabulary, presented to the judge of the Court of First Instance of Iloilo an application for search warrant, the said Ramon Gayanilo stating in hisapplication; "That in the house of Chino Uy Kheytin, Sto. Nio St., No. 20, Iloilo, under the writing desk inhis store, there is kept a certain amount of opium."- Armed with that search warrant, the respondent M. S. Torralba, on the same day (April 30th) searchedthe house of the petitioner Uy Kheytin and found therein 60 small cans of opium. They wanted to searchalso the bodega on the ground-floor of the house, but Uy Kheytin positively denied that it was his or thathe rented it. Lt. Torralba wanted to be sure, and for this reason, he placed a guard in the premises to seethat nothing was removed therefrom, and then went away to find out who the owner of the bodega was. The next morning he learned from the owner of the house, one Segovia, of the town of Molo, that theChinaman Uy Kheytin was the one who was renting the bodega. Thereupon Lt. Torralba and hissubordinates resumed the search and then and there found and seized articles which were all withconnection to the using of opium.- A criminal complaint was filed in the court of the justice of the peace of Iloilo against all the petitionersherein, charging them with a violation of the Opium Law. They were duly arrested.-Defendants urged: (1) that the search warrant of April 30 was illegal because the requisites prescribed bythe General Orders No. 58 had not been complied with in its issuance (specifically (a) because it was not issued upon either of the grounds mentioned in section 96 of General Orders No. 58, and (b) because the judge who issued it did not determine the probable cause by examining witnesses under oath) ;

(2) that the

7 searches and seizures made on May 1st had been made without any semblance of authority and henceillegal; and (3) that the seizure of the defendants' books and letters was a violation of the provisions of the Jones Law providing that no person shall be compelled to testify against himself, and protecting himagainst unreasonable searches and seizures. Issue: WON the defendants positions are with merit.

Ruling (s): 1. That although in the issuance of the search warrant in question the judge did not comply with therequirements of section 98 of General Orders No. 58, the petitioners are not entitled to the return of theopium and its paraphernalia which were found and seized under said warrant, and much less are theyentitled to be exonerated because of such omission of the judge.2. That the search made on May 1st was a continuation of the search begun on the previous day, and,therefore, did not require another search warrant.3. That the seizure of the petitioner's books, letters, telegrams, and other articles which have no inherentrelation with opium and the possession of which is not forbidden by law, was illegal and in violation of thepetitioners' constitutional rights. RD: (Contention # 1)-SEC. 96. of General Orders No. 58 provide: It (a search warrant) may be issued upon either of thefollowing grounds: 1. When the property was stolen or embezzled. ; 2. When it was used or when the intentexists to use it as the means of committing a felony.-Suffice it to say that, whatever may be the technical common-law meaning of the word "felony", which isused in paragraph 2 of sec. 96 above quoted, the Court believes it would be the height of absurdity tohold, upon technical grounds, that a search warrant is illegal which is issued to search for and seizeproperty the very possession of which is forbidden by law and constitutes a crime. Opium is such property.- Search-warrants have heretofore been allowed to search for material so kept as to endanger the publicsafety.- A search warrant may be likened to a warrant of arrest. The issuance of both is restricted by the sameprovision of the Jones Law (sec. 3) which is as follows: That no warrant shall issue but upon probablecause, supported by oath or affirmation, and particularly describing the place to be searched and the person or thing to be seized. - In the present case there was an irregularity in the issuance of the search warrant in question in that the judge did not first examine the complainant or any witnesses under oath. But the property sought to besearched for and seized having been actually found in the place described by the complainant, reasoningby analogy from the case of an improper arrest, we are of the opinion that that irregularity is not sufficientcause for ordering the return of the opium found and seized under said warrant, to the petitioners, andexonerating the latter.(Contention # 2)- While it is true that a warrant is good for 10 days after the date of issuance, this cannot be interpreted tomean that a search warrant can be used every day for 10 days, and for a different purpose each day. Thiswould be absurd.

8 -It appears from the oral evidence adduced during the hearing of the petitioners' motion in the court belowthat the search for opium, the property mentioned in the warrant, was not completed on April 30th; it wasinterrupted by the necessity to ascertain who the owner of the bodega on the ground-floor was, becausethe petitioner Uy Kheytin falsely disclaimed ownership thereof. In other words, the search of May 1st wasnot made "for a different purpose," nor could it be considered "another search," but was really acontinuation of the search begun on April 30th. This is shown by the fact that during the interval betweenthe two searches the premises in question were guarded by Constabulary soldiers, and the petitionerswere made to understand on April 30th that the authorities were not yet through with the search andwould continue the same as soon as they found out that the bodega was also occupied by the petitionerUy Kheytin.(Contention # 3)- In order to comply with the constitutional provisions regulating the issuance of search warrants, theproperty to be seized under a warrant must be particularly described therein and no other property can betaken thereunder.- That the officers of the law believed that the books, papers, etc., which they seized might be used asevidence against the petitioners herein a criminal action against them for a violation of the Opium Law, isno reason or

justification under the law for the seizure: First, because they were not "particularlydescribed" or even mentioned in the search warrant; second, because, even if they had been mentioned inthe search warrant, they could not be legally seized, for a search warrant cannot be used for the purposeof obtaining evidence; and third, because to compel a person to produce his private papers to be used inevidence against him would be equivalent to compelling him to be a witness against himself.

Burgos v. Chief of Staff, supra 6. Admissibility of Illegally-Seized Evidence Stonehill v. Diokno, supra Alih v. Castro, supra Pp v. Exala, 221 SCRA 494

I. THE FACTS A private jeep driven by accused-appellant Bocalan was stopped at a police checkpoint in Cavite City for routine inspection. With Bocalan were his co-accused Fernandez and Exala. Pfc. Galang, a member of the inspection team, went near the jeep and asked the occupants if there were firearms inside. They answered in the negative. Pfc. Galang proceeded to inspect the vehicle by beaming a flashlight inside. He then noticed a black leather bag measuring about 1 foot wide and 2 feet long with its sides bulging. When he asked what it contained, there was deadening silence from the 3 accused. Nobody answered. Instead, they suddenly became fidgety. Suspicious, Pfc. Galang ordered the bag opened, which was found out to contain marijuana. The 3 accused were thereafter prosecuted and convicted of illegal transportation of marijuana. Accused Bocalan appealed and questioned the legality of the admission of the marijuana as evidence against him since it was seized without a valid search warrant. II. THE ISSUE Was the marijuana seized without warrant during the checkpoint admissible in evidence against the accused? III. THE RULING [The 1st Division voted 3-1 to AFFIRM the conviction of the accused. Justices Grio-Aquino and Quiason concurred with Justice Bellosillos ponencia. Justice Cruz, by his lonesome, dissented from the majority.] The Court held that Bocalan is deemed to have waived his objection to the admission of the seized marijuana because he neither raised this issue before the trial court nor objected to the admissibility of the marijuana when it was offered in evidence. And even assuming that there was no such waiver, the Court held that still Bocalan s contention deserves scant consideration because there are instances where search and seizure can be made without necessarily being preceded by an arrest. An illustration would be the stop -and-search without a warrant at military or police checkpoints, the constitutionality of which has already been upheld by this Court [in Valmonte vs. De Villa]. Vehicles are generally allowed to pass through these checkpoints after a routine inspection and answering a few questions. If vehicles are stopped and extensively searched it is because of some probable cause which justifies a reasonable belief of those manning the checkpoints that either the

motorist is a law-offender or the contents of the vehicle are or have been instruments in the commission of an offense. According to the Court, lest it be misunderstood, the foregoing doctrine is not intended to do away with the general rule that no person shall be subjected to search of his person, personal effects and belongings, or his residence except of virtue of a search warrant or on the occasion of a lawful arrest. This case, however, is an incident to or an offshoot of a lawful stop -and-search at a military or police checkpoint. The checkpoint in the instant case was established in line with Operational Bakal, the main object of which was to search for unlicensed firearms and other prohibited items in the possession of unauthorized persons passing through it. When the jeep carrying the contraband passed through the checkpoint, it was flagged down and the occupants were asked routine questions. In the course thereof, Pfc. Galang noticed a black leather bag the sides of which were bulging. He asked what the contents of the bag were. None of the accused answered. At that moment, the demeanor of the accused changed; they became suspiciously quiet and nervous as if they were concealing something from Pfc. Galang. The accused clearly appeared to be in abject fear of being discovered. Such peculiar apprehensiveness if not restrained reaction of the accused, which did not appear normal, provided the probable cause justifying a more extensive search that led to the opening of the bag and the discovery of the prohibited stuff. [NOTE: Incidentally, one of the co-counsels for accused-appellant Bocalan in his appeal to the Supreme Court was then-Atty. and now Supreme Court Senior Associate Justice Presbitero Velasco Jr.]

CRUZ, J., dissenting: Justice Cruz maintained the proposition in his dissent in Valmonte vs. De Villa that checkpoints and the searches and seizures incident thereto are unconstitutional. In People vs. Exala, he expounded on this thesis:
I am opposed to checkpoints as regular police measures aimed at reducing criminality in general. I do not agree that in the interest of peace and order, any or every vehicle may be stopped at any time by the authorities and searched without warrant on the chance that it may be carrying prohibited articles. That possibility is not the probable cause envisioned in the Bill of Rights. In the case of the ordinary checkpoint, there is not even any suspicion to justify the search. The search is made as a matter of course, either of all vehicles or at random. There is no showing that a crime is about to be committed, is actually being committed, or has just been committed and the searching officer has personal knowledge that the person being searched or arrested is the culprit. I will concede that checkpoints may be established at borders of states or at constructive borders ne ar the boundary for the purpose of preventing violations of immigration and customs laws. But in the interior of the territory, the requirements of a valid search and seizure must be strictly observed. The only permissible exemption is where a crime like a bank robbery has just been committed or a jailbreak has just occurred, and the authorities have to seal off all possible avenues of escape in the area. In all other cases, I submit that the checkpoint should not be allowed. xxx. [W]e cannot retroactively validate an illegal search on the justification that, after all, the articles seized are illegal. That is putting the cart before the horse. I would rather see some criminals go unpunished now and then than agree to the Bill of Rights being systematically ignored in the oppressive checkpoint. Respect for the Constitution is more important than securing a conviction based on a violation of the rights of the accused. (Emphasis supplied.)

Pp v. Zaspa, 340 SCRA 752

DECISION

PANGANIBAN, J.: Arrest and prosecution in cases of illegal sale of prohibited drugs would be unduly impeded, unless evidence validly obtained during buy-bust operations conducted by police officers is admitted. Many times, the only way to expose and prosecute crimes involving prohibited drugs is through such operations. While this Court is ever vigilant in protecting and upholding the constitutional rights of the accused, it is also keenly aware of the need to rid society of this drug plague. Thus, after all the elements of a legitimate buy-bust operation have been duly proven by the prosecution, the Court will not hesitate to affirm a conviction. That is, unless the appellants can prove by competent and credible evidence that the operation was a sham or that their constitutional rights were violated or ignored. The Case Mila Razul and Roma Razul appeal the December 14, 2000 Decision1[1] of the Regional Trial Court (RTC) of Quezon City (Branch 103) in Criminal Case No. Q-98-80043. The dispositive portion of the Decision reads as follows: ACCORDINGLY, the court finds accused MILA RAZUL y Bashied and ROMA (or RONA) RAZUL y Bashied GUILTY beyond reasonable doubt as co-conspirators in the illegal selling or delivery of 212.15 grams of shabu in violation of Section 15, RA 6425, as amended, and they are hereby each sentenced to suffer the penalty of RECLUSION PERPETUA and to pay a fine of Ten Million Pesos.2[2] In the original Information3[3] dated December 8, 1998, appellants were charged, together with Bairona Bangke y Sarip, with violation of Section 15 of Article III in relation to Section 2 (e), (f), (m) and (o) of Article I of RA 6425 as amended by RA 7659. However, the charge against Bangke was later withdrawn and a new Information4[4] dated April 15, 1999, was filed against her for violation of Section 16 of RA 6425. The charge, to which she pleaded guilty with the assistance of counsel de parte,5[5] involved illegal possession of 5.29 grams of shabu.

Mila Razul and Roma Razul were indicted anew in an Information6[6] dated April 15, 1999, charging them as follows: That on or about the 6th day of December, 1998 in Quezon City, Philippines, the said accused, conspiring and confederating with and mutually helping one another not having been authorized by law to sell, dispense, deliver, transport or distribute any regulated drug, did then and there willfully and unlawfully sell or offer for sale 212.15 (Two Hundred Twelve point Fifteen) Grams of white crystalline substance containing Methylamphetamine Hydrochloride which is a regulated drug.7[7] During the arraignment of appellants on November 26, 1999, after the Information had been read and explained to them in Tagalog, a dialect that they fully understood,8[8] they pleaded not guilty.9[9] After pretrial and trial on the merits, they were found guilty of violating Section 15 of RA 6425. The Facts Version of the Prosecution In its Brief,10[10] the Office of the Solicitor General (OSG) presents the prosecutions version of the facts as follows: On December 6, 1998, SPO2 Wilfredo Red, SPO2 Brigido Juan, SPO1 Levy Sevilla, SPO1 Edgardo Buluran, Inspector Levy Sevilla and an informant named Akong underwent a briefing headed by Inspector Ratuita at the Central Police District, Station I, Mapua Street, La Loma, Quezon City in connection with drug bust operation against three suspected Muslims. SPO2 Wilfredo Red prepared the buy bust money consisting of two bundles of bond paper wrapped with pieces of one thousand peso bills, placed inside a boodle bag. About 11:30 in the morning of December 6, 1998, SPO2 Wilfredo Red rode with the informant in a Black Toyota Celica with Plate No. DFA 444, while the rest of the composite team rode in a Ford Fierra back-up vehicle, heading to the corner of Quirino Highway and Baesa Road, Quezon City. SPO2 Red posed as poseur buyer and waited for the accused in front of a gasoline station along Quirino Highway.

About 1:15 in the afternoon, SPO2 Red saw a taxicab coming from the direction of Balintawak towards Novaliches, which stopped in front of the gas station. Three (3) persons alighted from the cab. The informant Akong briefly introduced SPO2 Red to appellant Mila B. Razul. Thereupon, the informant told appellant Mila B. Razul, ito yong buyer natin ng stuff. SPO2 Red asked for P200,000.00 worth of shabu. At that instance, appellant Mila B. Razul took a blue plastic bag from another plastic bag in possession of Roma B. Razul and handed to SPO2 Red four (4) plastic packets containing white crystalline substance. Upon examination, SPO2 Red handed to Roma B. Razul the boodle bag containing the buy bust money who, in turn, placed the same inside her red bag. During the occasion, Bairona S. Bangke, a companion of appellant took out from the pocket of her pants a small sachet of the white substance and handed it to SPO2 Red as a give away item. The latter proceeded in raising his hand as a pre-arranged signal, prompting the back-up team to proceed with the arrest and apprehension of appellants and Bairona S. Bangke. On December 7, 1998, a chemistry examination was made on the confiscated substance which was found with positive results to the test for Methylamphetamine hydrocloride in the aggregrate weight of 207.15 grams.11[11] (Citations omitted) Version of the Defense On the other hand, appellants deny the allegations of the police officers. Their statement of facts, as contained in their 10-page Appeal Brief,12[12] is reproduced as follows: In their defense, appellants denied the allegations of the police officers. They a lleged that they accompanied Bairona Ban*g+ke in going to Baesa road to find the latters customer, a certain Ernie and his wife Cecile. While traveling on board a taxi, Bairona spotted Ernie at the gasoline station. Bairona stopped the taxi and alighted. She talked to Ernie and moment[s] later, Bairona asked appellant to transfer to the car of Ernie, as they would proceed to the house of Ernie. While on their way to the house of Ernie, an army type jeep blocked their way. The passengers of the jeep immediately alighted and poked their guns at the appellants. The two armed men boarded the car and one of them took the driver seat, while Ernie sat between the two armed men. When the car started running, the two armed men talked to Ernie as if they knew with [sic] each other. Thereafter, they allowed Ernie to alight. These two armed men were identified later as SPO1 Red and SPO1 Buluran. SPO1 Red poked his gun at the trio and told them that they are pushers and its nice to rape Muslims. When SPO1 Red continuously poking and swaying his gun, Rona held the arm of SPO1 Red and the gun suddenly fired that hit the foot of Mila. Thereafter, SPO1 Buluran who was then driving, stopped the car and stabbed Rona several times but hit her 4 times only in her stomach. Fortunately, a police mobile car arrived and stopped beside the car on which the appellants were on board as the appellants were screaming for help. When the occupants of the mobile car were about to respond, SPO1 Red and SPO1 Buluran shouted that they [were] also policemen. As the occupants of the mobile car approached and saw Mila and Rona both injured and bleeding, they immediately boarded the two into their mobile car and brought them to East Avenue Medical Center for

treatment. After [m]edical treatment, appellants were brought to the police station where they were surprised to know that they are being charged of violation of Sec. 15 of RA 6425.13[13] Trial Courts Ruling The RTC ruled that the prosecution evidence proved the guilt of appellants beyond reasonable doubt. It gave superior weight to the testimonies of the arresting police officers and the poseur-buyer, because these were consistent, sincere and could not have simply been rehearsed or scripted.14[14] Furthermore, the presumption of regularity in the performance of official functions was unrebutted by the defense. 15[15] On the other hand, appellants presented no evidence showing any motive or reason for the police officers to testify falsely against them. Hence, this appeal. The Issues Appellants raise the following alleged errors for this Courts consideration: 1. The court a quo erred in holding that there was a buy bust operation conducted against accused/appellants. 2. The court a quo erred in holding that the accused are guilty beyond reasonable doubt of the crime of selling 212.15 grams of shabu. The Courts Ruling The appeal is unmeritorious.

You might also like